Подсчет Порядок и последовательность Разрядное значение, нечетное и четное время сложения и вычитания Таблицы времени Умножение и деление Денежные фигуры Меры Обработка данных Решение проблем
Математические игры могут стать интересным способом изучения таблицы умножения и деления. Эта подборка математических игр была выбрана для того, чтобы научить детей, которые только начинают понимать принципы умножения и деления чисел. Они обеспечивают повторение, необходимое детям для закрепления их знаний о фактах умножения и деления.
Нажмите кнопку
Прекрасно подходит для оттачивания ваших математических навыков в уме с помощью таблиц умножения. Есть два варианта: «Нажми на ответ» или «Найди вопрос» на время! Подходит для планшетов.
Поезд ментальной математики
Mental Maths Train — это математическая игра, в которой основное внимание уделяется базовому словарному запасу сложения, вычитания, умножения и деления. Игра с множественным выбором, которая может дать детям уверенность в четырех арифметических операциях.
Кокосовые хлопья
Coconut Multiples может укрепить знания детей о таблицах умножения, помогая им распознавать кратные из каждой таблицы. Выберите кратные из таблицы умножения до 10 или 12.
Число Фактов Семей
Сможете ли вы найти все семейства числовых фактов? Как раз то, что нужно для изучения связи между сложением/вычитанием и умножением/делением.
Ужины собачьего подразделения
Разделите кости между милыми щенками. Этот восхитительный сайт представляет собой полезное введение в деление.
Диаграмма Венна
Ресурс с обучающим демонстрационным режимом и областью, где дети могут практиковать свои навыки сортировки кратных на диаграмме Венна.
Веселая мумия
Funky Mummy — это игра с фактами. Практикуйте быстрое запоминание фактов сложения, вычитания и умножения.
Дисплей массива
Действительно полезный обучающий инструмент для моделирования умножения и связи между умножением и делением.
Математика Рыбалка
Практикуйте свои навыки умножения. Игра охватывает не только таблицу умножения, но и удвоение, возведение в квадрат и куб чисел.
Умножение метеоров
Стреляйте в метеоры, в которых есть ответы на вопросы на умножение. Это действительно проверит ваши знания всех таблиц умножения! Есть три уровня сложности.
Отдел сноса
Игра, которая поможет вам научиться делению. Танки с проблемами дивизии движутся к вашему бластеру. Стреляйте по правильной проблеме, чтобы спасти свой бластер от разрушения. Хорошо для оттачивания умственных математических навыков.
Честные акции
Простая игра, которая знакомит с идеей деления небольших сумм поровну. Вы должны разделить лакомства между собаками, чтобы у них было поровну.
Десять кадров
Очень универсальный обучающий инструмент, который имеет 5, 10, 20 и 100 рамок или сеток для создания собственных рамок. Его можно использовать для сложения, вычитания, а также для создания массивов умножения.
Математика Чоппити Чоп
Эта игра охватывает ряд математических вопросов, таких как сложение, вычитание, умножение, деление, удвоение и многое другое. Ответьте на вопросы, затем нарежьте овощи, но избегайте стального стержня и пилы.
Словесные задачи на умножение и деление
Умножение и деление являются частью учебной программы по математике начальной школы . Это универсальные математические операции, которые закладывают основу для Количественных Способностей человека. Они являются основой почти всех математических понятий, встроенных в базовую арифметику , Геометрия , Алгебра и исчисление. Таким образом, важно, чтобы все учащиеся справились со словесными задачами на умножение и деление. Давайте узнаем больше о задачах на умножение и деление в этом блоге!
Этот блог включает в себя:
Что такое задачи на умножение и деление слов?
Задачи на умножение
Задачи на деление
Практические задачи
Часто задаваемые вопросы
Вопросы о кровном родстве Для конкурсных экзаменов
Вопросы о соотношении и пропорции
Что такое задачи на умножение и деление слов?
Словесные задачи на умножение и деление — это комплексные математические вопросы, разработанные, чтобы помочь учащимся освоить эти навыки, предлагая им сценарии из реальной жизни. Эти словесные задачи помогают маленьким детям относиться к проблемам на личном уровне, лучше активизируя их познание. Однако некоторые из них могут столкнуться с небольшими трудностями в понимании этих вопросов. Итак, давайте попробуем понять, как продвигаться с такими вопросами.
Попробуйте решить эти текстовые задачи, используя концепцию БОДМАС. Чтобы узнать больше об этом, прочитайте наш блог на BODMAS вопросы !
Задачи на умножение
Задачи на умножение обычно включают такие ключевые слова, как вычисление «общего» количества предметов, товаров или продукции. Вот несколько примеров задач на умножение:
В1: В классе 4 ряда сидений. В каждом ряду по 12 скамеек. Сколько учеников может сидеть в классе?
Ответ: В таких вопросах мы делаем следующие шаги:
Обратите внимание на количество скамеек в одном ряду = 12 Количество рядов = 4 Умножьте число на количество рядов = 12×4
Общее количество мест, доступных для студентов = 48
Таким образом, в классе могут сидеть 48 студентов!
Q2: Бутылочная фабрика производит 684 бутылки в день. Сколько бутылок произведет завод за 46 дней?
Ответ: Количество шин, производимых шинным заводом за один день = 684 Количество дней = 46
Общее количество произведенных бутылок = 684 x 46 = 31464
Следовательно, компания произведет 31464 бутылки за 46 дней .
Q3: В коробке 20 упаковок печенья. В каждой пачке 12 печений. Сколько печенья можно упаковать в 45 коробок?
Количество упаковок в 1 коробке = 20
Количество печенья в 1 упаковке = 12
Количество печенья в 1 коробке или 20 пакетах = 12 × 20 = 240
Количество печенья в 45 коробках = 240 × 45 = 10800
Таким образом, всего имеется 10 800 печений.
Q4: Месячная зарплата мужчины составляет рупий. 2500. Каков его годовой доход по зарплате?
Количество месяцев в году = 12
Заработная плата за 1 месяц = 2500 рупий
Годовая заработная плата = заработная плата за 12 месяцев = 2500 × 12 = рупий. 30 000
Следовательно, годовой доход = 30 000 рупий
Умножение-Задачи-Слова-Рабочий лист Загрузить
Задачи на деление
С другой стороны, задачи на деление, как правило, являются более сложными среди задач на умножение и деление. У них есть вопросы, связанные с разделением/помещением/сортировкой объектов в разные разделы. Вот несколько примеров, которые помогут вам лучше понять:
В1: В миске 140 арахиса. Рабочий должен положить их поровну в 4 пакета. Сколько арахиса будет в одном пакете? Ответ:
В задачах на деление мы всегда начинаем с определения делимого и делителя.
Общее количество арахиса будет взято в качестве дивиденда. Это потому, что мы должны отсортировать их на более мелкие группы, то есть разделить общее количество арахиса
Количество пакетов будет делителем, потому что мы должны разделить арахис на количество пакетов, которые у нас есть.
Общее количество арахиса, т. е. делимое = 140 Количество пакетов, т. е. делитель = 4 Количество пакетов = 140 / 4 После деления в частном получается 35.
Следовательно, общее количество арахиса в пакете будет 35.
Q2: В школе 241 2 студентов всего. Они помещают только 36 студентов в один класс. Сколько всего классов?
Ответ: Для этого вопроса мы также сначала определим делимое и делитель.
В вопросах деления для начинающих делимое можно определить, просмотрев вопрос в поисках наибольшего числа. Здесь общее количество студентов является самым большим числом. Кроме того, для размещения учащихся в классах необходимо разделить общее количество учащихся. Следовательно, общее количество студентов будет дивидендом, т.е. 2412
Количество учеников, которых нужно поместить в один класс, будет делителем в этом вопросе, так как это количество учеников, которых нужно поместить в класс. Итак, из 2412 студентов нам нужно сформировать группы по 36 человек. Затем мы выполняем базовую операцию деления
После выполнения деления мы получаем частное 67. Остаток равен 0. Таким образом, мы создадим 67 классов, чтобы вместить всех учащихся школы. Если бы остаток не был равен нулю, нам пришлось бы создать еще один класс, чтобы разместить дополнительных учеников.
Следовательно, общее количество классов будет 67.
Q3. рупий 5500 человек распределены поровну между 25 мужчинами. Сколько денег получит каждый?
Иллюстрации должны помочь вам понять, как решать задачи на умножение и деление. Вот несколько вопросов, основанных на той же концепции, для практики. Вопросы смешанные, поэтому вы также работаете над определением того, требует ли вопрос умножения или деления.
Q1: У Алекса есть 150 билетов на концерт. Он должен положить их в 10 коробок. Сколько билетов будет в одной коробке?
Q2: В упаковке 10 конфет. Владелец магазина покупает 124 упаковки этих конфет. Сколько всего конфет у него?
Q3: В пруду 5240 рыб. За месяц количество рыбы удваивается. Сколько рыбы будет в пруду через два месяца?
Q4: У работника есть 4500 отдельных листов. Он должен сделать тетради со 120 страницами. Сколько тетрадей он может сделать?
Q5: Изготовление карты занимает 15 минут. Рабочий работает 6 часов в день. Сколько карточек он сможет сделать?
Q6: Студент собирает листья для научного эксперимента. Он собирает 6 новых видов листьев за день. Сколько листьев у него будет через неделю?
Q7: Продавец зарабатывает 10$ на продаже. Если он совершит 123 продажи, сколько денег он сможет заработать?
Q8: Рон берет своих учеников на школьную экскурсию. В школе 2420 учащихся. Автобус может вместить только 45 студентов. Сколько автобусов ему понадобится, чтобы отвезти всех студентов в поездку?
Q9: Компания производит 12 луковиц в день. Они работают 6 дней в неделю. Сколько луковиц они смогут произвести через 8 недель?
Q10: Самьяк делает коробки для ручек в своей мастерской. Если он работает 3 часа, он может сделать 1 коробку. Он работает по 8 часов в день. Сколько коробок он сможет сделать за 12 дней?
Словесные задачи на умножение и деление — это комплексные математические вопросы, разработанные, чтобы помочь учащимся освоить эти навыки, предлагая им сценарии из реальной жизни.
Месячная зарплата мужчины составляет рупий. 2500. Каков его годовой доход по зарплате?
Количество месяцев в году = 12 Заработная плата за 1 месяц = 2500 рупий Годовая заработная плата = заработная плата за 12 месяцев = 2500 × 12 = рупий. 30 000 Следовательно, годовой доход = 30 000 рупий
рупий. 5500 человек распределены поровну между 25 мужчинами. Сколько денег получит каждый?
Итак, к концу этого блога мы дали вам все, что нужно было знать о задачах на умножение и деление. Надеюсь, у вас не возникло трудностей с пониманием концепций. Продолжайте практиковаться, и вы сможете быстро освоить тему. Чтобы помочь вам с такими темами, у нас есть много информативных блогов, которые ждут вас.
Украшает, форматирует и сделает CSS код более читаемым.
Уменьшитель CSS
Сделает CSS код уменьшенным, сжатым путем удаления новых строк, пробелов, комментариев и отступов.
Украшатель HTML
Украшает, форматирует и сделает HTML код более читаемым.
Уменьшитель HTML
Сделает HTML код уменьшенным, сжатым путем удаления новых строк, пробелов, комментариев и отступов.
Украшатель Javascript
Украшает, форматирует и сделает Javascript код более читаемым.
Уменьшитель Javascript
Сделает Javascript код уменьшенным, сжатым путем удаления новых строк, пробелов, комментариев и отступов.
Обфускатор Javascript
Сделает Javascript код более сложным для понимания или чтения для защиты.
Украшатель JSON
Украшает, форматирует и сделает JSON код более читаемым.
Уменьшитель JSON
Сделает JSON код уменьшенным, сжатым путем удаления новых строк, пробелов, комментариев и отступов.
Украшатель XML
Украшает, форматирует и сделает XML код более читаемым.
Уменьшитель XML
Сделает XML код уменьшенным, сжатым путем удаления новых строк, пробелов, комментариев и отступов.
Украшатель OPML
Украшает, форматирует и сделает OPML код более читаемым.
Уменьшитель OPML
Сделает OPML код уменьшенным, сжатым путем удаления новых строк, пробелов, комментариев и отступов.
Украшатель SQL
Украшает, форматирует и сделает SQL код более читаемым.
Уменьшитель SQL
Сделает SQL код уменьшенным, сжатым путем удаления новых строк, пробелов, комментариев и отступов.
Конвертеры
Конвертер CSV в JSON
Конвертирует CSV данные в JSON и украшает.
Конвертер CSV в TSV
Конвертирует CSV данные в TSV и украшает.
Конвертер CSV в Excel
Конвертирует CSV данные в Excel и украшает.
Конвертер CSV в HTML
Конвертирует CSV данные в HTML, просматривая его ниже.
Конвертер CSV в SQL
Конвертирует CSV в SQL формат и украшает.
Конвертер CSV в Многострочные данные
Конвертирует CSV в многострочные данные и делает его более читаемым.
Конвертер CSV в Текст
Конвертирует CSV в обычный текст и делает его более читаемым.
Конвертер CSV в XML/JSON
Конвертирует CSV в XML и JSON онлайн.
Конвертер CSV в XML
Конвертирует CSV в XML и украшает.
Конвертер CSV в YAML
Конвертирует CSV в YAML и украшает.
Извлечь столбец CSV
Извлекает один столбец из CSV.
Конвертер Excel в CSV
Конвертирует Excel в CSV и украшает.
Конвертер Excel в TSV
Конвертирует Excel в TSV и украшает.
Конвертер Excel в HTML
Конвертирует Excel в HTML и украшает.
Excel в формульный вид
Конвертирует Excel в формульный вид и украшает.
Конвертер Excel в SQL
Конвертирует Excel в SQL и украшает.
Конвертер Excel в JSON
Конвертирует Excel в JSON и украшает.
Конвертер Excel в XML
Конвертирует Excel в XML и украшает.
Конвертер Excel в YAML
Конвертирует Excel в YAML и украшает.
Конвертер Excel в Текст
Конвертирует Excel в Текст и украшает.
Извлечь столбец Excel
Извлекает один столбец из Excel.
Конвертер TSV в JSON
Конвертирует данные TSV в JSON и украшает.
Конвертер TSV в CSV
Конвертирует данные TSV в CSV и украшает.
Конвертер TSV в Excel
Конвертирует данные TSV в Excel и украшает.
Конвертер TSV в HTML
Конвертирует данные TSV в HTML, с просмотром ниже.
Конвертер TSV в SQL
Конвертирует TSV в SQL формат и украшает.
Конвертер TSV в Многострочные данные
Конвертирует TSV в многострочные данные и делает более читаемым.
Конвертер TSV в Текст
Конвертирует TSV в обычный текст и делает более читаемым.
Конвертер TSV в XML/JSON
Конвертирует TSV в XML/JSON и украшает.
Конвертер TSV в XML
Конвертирует TSV в XML и украшает.
Конвертер TSV в YAML
Конвертирует TSV в YAML и украшает.
Извлечь столбец TSV
Извлекает один столбец из TSV.
Конвертер HTML в CSV
Конвертирует HTML в CSV и украшает.
Конвертер HTML в EXCEL
Конвертирует HTML в EXCEL и украшает.
Конвертер HTML в TSV
Конвертирует HTML в TSV и украшает.
Конвертер HTML в Многострочные данные
Конвертирует HTML в Многострочные данные и украшает.
Конвертер HTML в JSON
Конвертирует HTML в JSON и украшает.
Конвертер HTML в XML
Конвертирует HTML в XML и украшает.
Конвертер HTML в YAML
Конвертирует HTML в TAML и украшает.
Конвертер HTML в SQL
Конвертирует HTML в SQL и украшает.
Конвертер HTML в PHP
Конвертирует HTML в PHP и украшает.
Конвертер HTML в Javascript
Конвертирует HTML в Javascript и украшает.
Конвертер HTML в Asp
Конвертирует HTML в Asp и украшает.
Конвертер HTML в JSP
Конвертирует HTML в JSP и украшает.
Конвертер HTML в Perl
Конвертирует HTML в Perl и украшает.
Конвертер HTML в Jade
Конвертирует HTML в Jade и украшает.
Конвертер HTML в Текст
Конвертирует HTML в обычный текст.
Конвертер Jade в HTML
Конвертирует Jade в HTML и украшает.
Конвертер Markdown в HTML
Конвертирует Markdown в HTML код.
Конвертер JSON в XML
Конвертирует JSON в XML и украшает.
Конвертер JSON в CSV
Конвертирует JSON в CSV и украшает.
Конвертер JSON в Excel
Конвертирует JSON в Excel и украшает.
Конвертер JSON в TSV
Конвертирует JSON to TSV и украшает.
Конвертер JSON в YAML
Конвертирует JSON в YAML и украшает.
Конвертер JSON в HTML
Конвертирует JSON в HTML и украшает.
Конвертер JSON в SQL
Конвертирует JSON в SQL и украшает.
Конвертер JSON в C# класс
Конвертирует JSON в C# класс и украшает.
Конвертер JSON в Текст
Конвертирует JSON в Текст и украшает.
Конвертер SQL в HTML
Конвертирует SQL в HTML и украшает.
Конвертер SQL в CSV
Конвертирует SQL в CSV и украшает.
Конвертер SQL в Excel
Конвертирует SQL в Excel и украшает.
Конвертер SQL в TSV
Конвертирует SQL в TSV и украшает.
Конвертер SQL в XML
Конвертирует SQL в XML и украшает.
Конвертер SQL в JSON
Конвертирует SQL в JSON и украшает.
Конвертер SQL в YAML
Конвертирует SQL в YAML и украшает.
Конвертер SQL в Text
Конвертирует SQL в Text и украшает.
Конвертер XML в JSON
Конвертирует XML в JSON и украшает.
Конвертер XML в CSV
Конвертирует XML в CSV и украшает.
Конвертер XML в Excel
Конвертирует XML в Excel и украшает.
Конвертер XML в TSV
Конвертирует XML в TSV и украшает.
Конвертер XML в YAML
Конвертирует XML в YAML и украшает.
Конвертер XML в HTML
Конвертирует XML в HTML и украшает.
Конвертер XML в SQL
Конвертирует XML в SQL и украшает.
Конвертер XML в Текст
Конвертирует XML в Текст и украшает.
Конвертер YAML в XML/JSON/CSV
Конвертирует YAML в JSON/CSV/XML и украшает.
Конвертер YAML в Excel
Конвертирует YAML в Excel и украшает.
Конвертер YAML в HTML
Конвертирует YAML в HTML и украшает.
Конвертер XML в PDF
Конвертировать XML в PDF и Скачать.
Конвертер CSV в PDF
Конвертировать CSV в PDF и Скачать.
Конвертер TSV в PDF
Конвертировать TSV в PDF и Скачать.
Конвертер EXCEL в PDF
Конвертировать EXCEL в PDF и Скачать.
Конвертер JSON в PDF
Конвертировать JSON в PDF и Скачать.
Конвертер YAML в PDF
Конвертировать YAML в PDF и Скачать.
Конвертер SQL в PDF
Конвертировать SQL в PDF и Скачать.
Конвертер Текст в PDF
Конвертировать Текст в PDF и Скачать.
Конвертер PDF в JPG
Конвертировать PDF в JPG и Скачать.
Конвертер PDF в PNG
Конвертирует PDF в PNG и украшает.
Конвертер Текст в HTML
Конвертирует Текст в HTML и украшает.
Конвертер RSS в JSON
Конвертирует RSS в JSON и украшает.
Конвертер OPML в JSON
Конвертирует OPML в JSON и украшает.
Инструменты проверки валидности кода
Валидатор CSS
Проверьте ваш исходник CSS.
Валидатор Javascript
Проверьте ваш исходник Javascript.
Тестер Javascript
Проверьте ваш Javascript код.
Тестер HTML
Проверьте ваш HTML код.
Валидатор JSON
Проверьте ваш JSON код и украсьте.
Валидатор XML
Проверьте ваш XML код и украсьте.
Валидатор YAML
Проверьте ваш YAML код и украсьте.
Валидатор UUID
Проверьте ваш UUID код.
Тестер XPath
Онлайн Xpath тестер.
Тестер и генератор регулярных выражений
Проверка регулярного выражения и создание кода.
Препроцессоры CSS
Компилятор LESS
Создает отформатированные стили CSS из меньшего источника.
Компилятор Stylus
Создает украшенные стили CSS из Stylus
Конвертер CSS в LESS
Конвертирует CSS в Less и украшает.
Конвертер CSS в SCSS
Конвертирует CSS в SCSS и украшает.
Конвертер CSS в SASS
Конвертирует CSS в SASS и украшает.
Другие утилиты
Генераторы
Генератор случайных паролей
Генератор Favicon
Безопасный каталог htaccess
Генератор htpasswd
Генератор Lorem Ipsum
Генератор адресов IPv4
Генератор адресов IPv6
Генератор MAC адресов
Генератор календарных дат
Конвертеры величин
Конвертер веса
Конвертер площади
Конвертер плотности и массы
Конвертер байтов/битов
Конвертер электроэнергии
Конвертер энергии
Конвертер силы
Конвертер Топлива
Конвертер длины
Конвертер объема и емкости
Конвертер температуры
Конвертер скорости и ускорения
Конвертер угла
Конвертер массы
Конвертер мощности
Конвертер давления и напряжения
Конвертер времени
Астрономический конвертер
Конвертер частоты
Утилиты
Информация о браузере
Конвертер Base64 в Изображение
Конвертер Изображение в Base64
Конвертер Файла в Base64
Генератор символов
Конвертер текста в HTML объекты
Парсер URL
Автообновление страницы
Экранирование и разэкранирование
Экранирование и разэкранирование JSON
Экранирование и разэкранирование C#
Экранирование и разэкранирование Javascript
Экранирование и разэкранирование Java
Экранирование и разэкранирование CSV
Экранирование и разэкранирование SQL
Экранирование и разэкранирование HTML
Экранирование и разэкранирование XML
Шифрование
Генератор HMAC
Хэш калькулятор
Стеганография изображений
Стеганографический декодер
Генератор паролей MySQL/MariaDB
Генератор паролей Postgres
Строчные утилиты
Конвертер базового номера
Кодер/Декодер Base64
Средство просмотра различий
Кодировщик Url
Декодер Url
Кодировщик Html
Декодер Html
Добавить слэш
Убрать слеш
Конвертер числа в слово
Утилиты строк
Трансформер текста
Конвертер регистра
Калькулятор даты
Конвертер Даты/Времени в временную метку Unix
Конвертер временную метку Unix в время Дата/Время
Конвертер Секунд в человеческое время
Конвертер Секунд в Часы:Минуты:Секунды
Конверторы изображений
Конвертер JPG в PNG
Конвертер PNG в JPG
Конвертер GIF в PNG
Конвертер PNG в GIF
Конвертер BMP в PNG
Конвертер BMP в JPG
Генератор изображений с закругленными углами
Инструменты домена и IP
Получить IP и имя хоста
Просмотр имени хоста
Whois сервис
Просмотр DNS
Просмотр MX
Просмотр сервера имён
Проверка IP сайта
IP утилиты
Мой IP адрес
Редакторы кода
Просмотр исходного кода
Онлайн Редактор Кода
Пример кода
Конвертеры цвета
Конвертер RGB в HEX
Конвертер RGB в CMYK
Конвертер RGB в HSV
Конвертер HEX в HSV
Конвертер HEX в CMYK
Конвертер HSV в CMYK
конвертировать файлы Excel в PDF онлайн — лучший конвертер XLS в PDF
конвертировать файлы Excel в PDF онлайн — лучший конвертер XLS в PDF
Loading. ..
Конвертируйте электронную таблицу Excel в PDF онлайн и сделайте ее читабельной!
Конвертер Excel в PDF
Конвертируйте электронную таблицу Excel в PDF онлайн и сделайте ее читабельной!
Дополнительные инструменты от MergePDF.io
Кто может использовать конвертер Excel в PDF?
Авторы контента могут использовать конвертер XLS в PDF для преобразования своего титульного листа в формат PDF
Студенты-финансисты могут использовать этот инструмент для преобразования своей электронной таблицы по финансам в формат PDF.
Менеджеры по персоналу могут извлечь выгоду из этого конвертера Excel в PDF для хранения зарплатных ведомостей и других документов.
Корпоративные специалисты могут воспользоваться этим инструментом, чтобы придать своим документам профессиональный вид.
Исследователи могут использовать этот инструмент для преобразования своего статистического анализа в файлы PDF.
Как конвертировать Excel в PDF
Чтобы преобразовать Excel в PDF онлайн, выполните следующие действия:
1
Загрузить файл Excel
Сначала нажмите кнопку «Загрузить файл» и загрузите электронную таблицу Excel, которую вы хотите преобразовать. Наш онлайн-конвертер Excel в PDF также поддерживает функцию перетаскивания.
Кроме того, вы также можете загружать файлы из облачных сервисов хранения, таких как Dropbox и Google Drive.
2
Конвертировать и скачать
Наш конвертер Excel в PDF мгновенно преобразует файлы в формате Excel в документ PDF в мгновение ока.
Преобразованный PDF-файл будет готов к загрузке через несколько секунд.
Нажмите кнопку «Скачать» и сохраните PDF-файл на свой компьютер.
3
Приступить к другим функциям
После преобразования документа из Excel в PDF есть другие замечательные инструменты, с помощью которых можно объединять PDF-файлы, разделять, сжимать и преобразовывать их в другие форматы.
Функции
Эффективно работает на всех платформах
Этот онлайн-конвертер Excel в PDF работает на всех: устройствах (ноутбуки, настольные компьютеры и мобильные телефоны) браузерах (Google Chrome, Safari, Opera и Firefox) операционных системах (Windows, Android, Linux и Mac)
Сверхбыстрый процесс преобразования
Преобразование XLS в PDF никогда не было легкой задачей до появления этого умного инструмента! Наш инструмент может конвертировать Excel в PDF за пару секунд, используя наш онлайн-конвертер Excel в PDF.
Защищает вашу конфиденциальность
Вам неудобно загружать конфиденциальный файл Excel? Не беспокойся! С нашим инструментом преобразования Excel в PDF вам не нужно об этом беспокоиться. Сервер нашего инструмента автоматически стирает все ваши файлы. Поэтому ваши данные остаются на 100% в безопасности.
Сохраняет исходное качество
Когда вы используете наш инструмент для преобразования XLS в PDF, он в значительной степени сохраняет формат файла Excel. Общее количество столбцов, строк и таблиц будет таким же, как и в документе PDF. Таким образом, вам не нужно редактировать позже
Облачная обработка
Чем больше программ вы запускаете на своем устройстве, тем медленнее оно будет работать. К счастью, этого не произойдет, если вы используете наш онлайн-конвертер Excel в PDF. Ваши файлы Excel будут преобразованы в PDF в облаке. Таким образом, наш инструмент не занимает место на вашем устройстве.
Абсолютно бесплатно
Наш конвертер Excel в PDF можно использовать на 100% бесплатно. Таким образом, вам не нужно платить за использование этого онлайн-конвертера. Пользовательских ограничений нет. Вы можете использовать этот конвертер XLS в PDF столько раз, сколько захотите.
Почему наш конвертер Excel в PDF лучший
Благодаря удобному интерфейсу им довольно легко пользоваться.
Он мгновенно преобразует все электронные таблицы Excel в файлы PDF.
Все ваши файлы XLS в PDF будут автоматически удалены с сервера.
Этот конвертер Excel в PDF доступен на нескольких языках, таких как испанский, португальский, французский и другие.
Есть вопрос?
С нашим онлайн-конвертером Excel в PDF вы можете легко преобразовать все свои электронные таблицы XLS в PDF, не потратив ни копейки. Все, что вам нужно сделать, это загрузить файл Excel в поле ввода и нажать кнопку «Конвертировать». Ваш файл будет преобразован в PDF за считанные секунды.
После того, как вы преобразовали Excel в файл PDF с помощью нашего инструмента, вы можете нажать кнопку «Загрузить», чтобы сохранить документ PDF.
Наш онлайн-конвертер XLS в PDF поддерживает все операционные системы, включая Linux, Mac, Android и Windows. Кроме того, он совместим со всеми устройствами (мобильными телефонами, ноутбуками и настольными компьютерами).
Вам не нужно покупать какое-либо платное программное обеспечение. Вместо этого вам нужно только стабильное подключение к Интернету. Самое главное, вы можете использовать этот инструмент преобразования Excel в PDF столько раз, сколько захотите.
Да, вы можете использовать этот инструмент на своем мобильном телефоне. Наш онлайн-конвертер Excel в PDF совместим со всеми мобильными телефонами, ноутбуками и настольными компьютерами.
Нам доверяют миллионы людей по всему миру
MergePDF.io позволяет пользователям объединять несколько файлов PDF в один, сохраняя при этом свои данные в безопасности. Используйте этот инструмент с умом и следите за всеми своими PDF-файлами, прежде чем вы потеряете их из виду!
Оцените этот инструмент
4.9 /
682,860
голосов
Как преобразовать Excel в PDF и уместить одну страницу
Использование MS Excel — сложная задача, особенно для пользователей, которые не владеют ею. Если вы являетесь постоянным пользователем Excel, то существует более высокая вероятность того, что вы столкнулись с проблемой, когда ваш лист Excel печатается на нескольких страницах, а не на одной. Это расстраивает, так как опубликовано всего несколько строк, а остальная часть бумаги будет потрачена впустую. Если вы по-прежнему часто сталкиваетесь с этой проблемой, прочтите этот блог. Мы покажем вам, как сохранить Excel в PDF и уместить его на одной странице.
Кроме того, ниже представлены советы по преобразованию Excel в PDF без обрезки.
В этой статье
Способ 1. Печать Excel в PDF с помощью Fit Sheet на одной странице
Способ 2. Преобразование Excel в PDF на одной странице
Способ 3. Преобразование Excel в PDF без разрывов страниц
Способ 4. Печать Несколько файлов Excel в PDF на одной странице
Советы: как конвертировать Excel в PDF без обрезки?
Печать из Excel в PDF путем вписывания листа на одной странице
Самый простой способ преобразовать Excel в PDF на одной странице — настроить параметр масштабирования на «По размеру одной страницы» перед печатью. Вот как это сделать.
Шаг 1 Выберите принтер Excel в PDF
Запустите Microsoft Excel и откройте лист Excel, который вы хотите сохранить в формате PDF и разместить на одной странице. Нажмите «Файл» > «Печать», чтобы получить доступ к настройкам печати. Выберите принтер Microsoft Print to PDF.
Если у вас установлен Wondershare PDFelement — PDF Editor, выберите PDFelement в качестве принтера. Преимущество печати Excel в PDF с помощью PDFelement заключается в том, что вы можете впоследствии редактировать преобразованные файлы PDF с помощью этого мощного редактора PDF, например, редактировать текст, настраивать макет страницы и т. д.
Попробуйте бесплатно
Попробуйте бесплатно
КУПИТЬ СЕЙЧАС
КУПИТЬ СЕЙЧАС
Шаг 2 Excel в PDF По размеру одной страницы
Чтобы уместить электронную таблицу Excel на одной странице, нажмите «Без масштабирования» в разделе «Настройки» и выберите «По размеру листа на одной странице»; теперь нажмите кнопку «Печать».
Появится диалоговое окно, и здесь вы должны ввести имя, выбрать место, где вы хотите сохранить файл, и нажать кнопку «Сохранить», чтобы распечатать Excel в PDF на одной странице.
Преобразование Excel в PDF на одной странице
Существует еще один способ сохранить электронную таблицу или книгу Excel на одной странице в Microsoft Excel. Это можно сделать с помощью параметра «Параметры страницы» на вкладке «Разметка страницы». Если вы не получали его, не волнуйтесь; вы можете следовать пошаговому руководству, представленному ниже.
Шаг 1 Параметры настройки открытия страницы
Предполагается, что вы уже открыли файл Excel с помощью Microsoft Excel. Теперь нажмите на вкладку «Макет страницы», и на этой вкладке вы должны щелкнуть крошечное средство запуска диалогового окна, появляющееся в правом нижнем углу. Это приведет к отображению диалогового окна «Параметры страницы».
Шаг 2 Вписать Excel в одну страницу
Щелкните вкладку Страница в диалоговом окне. Нажмите «Подгонка» под «Масштабирование» на вкладке «Страница» в диалоговом окне. Вы должны выбрать 1 в обоих полях рядом с опцией Fit to. Excel сократит все данные на одну страницу, и вы нажмете кнопку «ОК», чтобы сохранить настройку.
Шаг 3 Преобразование Excel в PDF на одной странице
Теперь нажмите «Файлы» > «Сохранить как» и сохраните файл Excel в формате PDF на своем компьютере.
Если вы по-прежнему не можете разместить все содержимое электронной таблицы на одной странице, следуйте способу 3, чтобы настроить разрыв страницы, чтобы устранить проблему.
Преобразование Excel в PDF без разрывов страниц
При печати с помощью Microsoft Excel программа автоматически устанавливает разрывы страниц, если содержимое листа разделено на разные страницы. В представлении макета страницы вы можете настроить места разрывов страниц, чтобы включить все, что вы хотите отобразить на одной странице.
Шаг 1. Открытие разрывов страниц
Перейдите на вкладку «Вид» и нажмите «Предварительный просмотр разрывов страниц». Добавленные автоматические разрывы страниц появятся на вашем листе.
Шаг 2 Настройка разрывов страниц
Вы можете переместить разрыв страницы, чтобы включить все на листе на одну страницу без разрыва страницы. Поместите курсор на синюю линию и перетащите его в нужном вам направлении. Отпустите, когда закончите. Содержимое внутри синих линий будет сохранено на одной странице.
Теперь вы можете перейти в «Файлы» > «Печать» и распечатать Excel на одной странице PDF.
Печать нескольких файлов Excel в PDF на одной странице
Если вы хотите преобразовать целую книгу с несколькими листами в одну страницу в формате PDF, этот вариант подойдет вам. В этом методе мы будем использовать Wondershare PDFelement — PDF Editor, известную программу просмотра и редактирования PDF для преобразования Excel в PDF без ущерба для качества или формата документа.
Он поставляется с инструментами редактирования, которые позволяют редактировать даже после преобразования файлов Excel в PDF-файлы. Вы также можете добавлять медиафайлы, такие как изображения и аннотации. Помимо преобразования, вы можете создавать редактируемые формы PDF, которые помогут пользователям получать информацию от клиентов в режиме реального времени, добавлять текст в PDF и многое другое.
Шаг 1 Загрузка и установка
Самым первым шагом, с которого начинается процесс, является установка PDFelement. Его просто загрузить, так как вы можете получить его по ссылке, указанной ниже, и установить его в соответствии с инструкциями, доступными в настройках установки.
Попробуйте бесплатно
Попробуйте бесплатно
КУПИТЬ СЕЙЧАС
КУПИТЬ СЕЙЧАС
Шаг 2 Загрузите файл Excel
Запустите PDFelement на своем компьютере и перейдите в главное окно. Щелкните значок «Создать PDF» и найдите файл Excel, который вы хотите преобразовать в PDF. Загрузите его и откройте.
Шаг 3 Настройка для Excel в PDF
Затем нажмите «Преобразовать» и перейдите в «Настройки печати». Здесь вы должны выбрать количество страниц на листе, чтобы добавить все рабочие листы на одну страницу. Затем нажмите «Сохранить», чтобы сохранить этот PDF-файл на своем компьютере.
Устали от старых медленных конвертеров PDF? Перейдите по ссылке ниже, чтобы узнать больше о том, как лучше всего работать с любым PDF-файлом.
Как преобразовать Excel в PDF без обрезки?
Иногда при попытке распечатать документ Excel в формате PDF отсутствует нижняя часть или поля страницы. Чтобы решить эту проблему, вы должны убедиться, что содержимое не обрезано разрывами страниц, и выбрать все, что вы хотите включить, чтобы настроить область печати. Если ничего из этого не помогает, вы можете попробовать следующие 2 совета.
1. Отрегулируйте качество страницы до 600 dpi
Некоторые пользователи решают проблему, увеличивая качество страницы до 600 dpi. Щелкните Файлы > Печать. Откройте настройки страницы и в параметре «Качество печати» установите значение 600 dpi
2.
Переключитесь на другой шрифт или измените размер шрифта.
Некоторые пользователи считают полезным изменить текст на другой шрифт или использовать меньший размер шрифта для экономии места. При наличии свободного места лист Excel можно сохранить в формате PDF без обрезки.
Онлайн-конвертер XLS (Excel) в PDF
Вертопал — Бесплатный онлайн конвертер
Перетащите файлы в любое место для загрузки
Дом
Документ
Конвертер XLS в PDF
Конвертируйте электронные таблицы XLS в формат PDF онлайн и бесплатно.
Преобразовать
XLS
к
PDF
Загрузка загрузчика. ..
Если вы загрузили файл, он будет отображаться.
Подтвердить
Отмена
Метки:
microsoft-office
превосходить
электронная книга
электронная таблица
Как преобразовать
XLS в PDF ?
1Загрузить
XLS Файл
Выберите любой документ электронной таблицы XLS , который вы хотите преобразовать, нажав кнопку «Выбрать файл».
2Select
XLS Инструменты
Используйте доступные инструменты для преобразования XLS в PDF и нажмите кнопку Convert.
3Загрузите файл
PDF
Подождите некоторое время для завершения преобразования PDF , а затем загрузите файл.
Инструменты
Шифрование
PDF
Преобразование XLS в защищенный паролем 9 012219 9 012219 9
Часто задаваемые вопросы
Как изменить формат XLS на PDF?
Чтобы изменить формат XLS на PDF, загрузите файл XLS, чтобы перейти на страницу предварительного просмотра. Используйте любые доступные инструменты, если вы хотите редактировать и манипулировать файлом XLS. Нажмите на кнопку преобразования и дождитесь завершения преобразования. После этого загрузите преобразованный PDF-файл.
Преобразование файлов на рабочем столе
MacOS
Windows
Linux
Преобразование
XLS в PDF на Macos979797777797979797 . Выполните следующие шаги, если вы установили Vertopal CLI в своей системе macOS.
Откройте терминал macOS.
Либо cd по XLS местоположение файла или укажите путь к входному файлу.
Вставьте и выполните приведенную ниже команду, заменив свое имя XLS_INPUT_FILE или путь.
$ конвертировать XLS_INPUT_FILE —в pdf
Преобразование
XLS по PDF для Windows
Следуйте приведенным ниже инструкциям, если вы установили Vertopal CLI в своей системе Windows.
Откройте командную строку или Windows PowerShell.
Либо cd по XLS местоположение файла или укажите путь к входному файлу.
Вставьте и выполните приведенную ниже команду, заменив свое имя XLS_INPUT_FILE или путь.
$ конвертировать XLS_INPUT_FILE —в pdf
Преобразование
XLS в PDF на Linux
Выполните следующие шаги, если вы установили Vertopal CLI в своей системе Linux.
1.2 Определители матриц второго и третьего порядка
1.3 Разложение определителя матрицы по элементами строки и столбца
1.4 Обратная матрица
1.1 Действия над матрицами
Прямоугольная таблица чисел, содержащая m строк и n столбцов, называется матрицей размера m*n:
Каждый элемент матрицы снабжается двумя индексами: первый указывает номер строки, второй – номер столбца, в которых расположен этот элемент.
Две матрицы называются равными, если числа их строк и столбцов равны и если равны элементы, расположенные на соответствующих местах этих матриц.
Если число столбцов матрицы n равно числу ее строк, то матрицу называют
квадратной матрицей порядка n. Элементы а11, а12,. .,ann квадратной матрицы порядка n образуют ее главную диагональ.
Квадратная матрица называется диагональной, если все ее элементы, расположенные вне главной диагонали, равны нулю.
Диагональная матрица Е называется единичной, если все ее элементы, расположенные на главной диагонали, равны единице.
Чтобы умножить матрицу на число, надо каждый элемент матрицы умножить на это число.
Суммой матриц A и B одинаковых размеров называется матрица, элементы которой равны суммам элементов матриц A и B, расположенных на соответствующих местах:
+=
Матрицу можно умножить на матрицу только в том случае, когда число столбцов матрицы равно числу строк матрицы . В результате умножения получится матрица
, у которой столько же строк, сколько их в матрице , и столько же столбцов, сколько их в матрице.
Элементы матрицы вычисляются по формуле
т. е. для получения элемента
, расположенного в i-й строке и j-м столбце матрицы c, надо элементы i-й строки матрицы A умножить
на соответствующие элементы j -го столбца матрицы B и полученные произведения
сложить.
Пример.
=
1.2 Определители матриц второго и третьего порядка
Определителем матрицы второго порядка называется число
Определителем матрицы третьего порядка называется число
1.3 Разложение определителя матрицы по элементами строки и столбца
МиноромMij элемента aij матрицы n-го порядка называется определитель (n–1)-го порядка,
который получается в результате вычеркивания в матрице n-го порядка строки и столбца, содержащих элемент
aij.
Алгебраическим дополнениемAij элемента
aij матрицы A называется его минор, умноженный на (–1)i+j:
Каждый определитель равен сумме произведений элементов любой его строки (столбца) на их алгебраические дополнения.
Пример 1. Вычислить определитель, разлагая его по элементам третьего столбца.
=+
++=
Пример 2. Найти минор элемента a44 в определителе четвертого порядка
Ответ:
1.4 Обратная матрица
Матрица A-1называется обратной для квадратной матрицы A , если
Квадратная матрица A имеет обратную тогда и только тогда, когда ее определитель не равен нулю.
Квадратная матрица A, определитель которой не равен нулю, имеет единственную обратную матрицу
где Δ– определитель матрицы ;
Aij– алгебраическое дополнение элемента матрицы .
Наверх
На главную
На следующую страницу
1.2. Определитель квадратной матрицы
Каждой квадратной матрице можно поставить
в соответствие число, вычисляемое по
определенному правилу. Если матрица
то
ее определителем называется число,
которое вычисляется по формуле:
(1.2.1.)
Например, для матрицы определитель
Рассмотрим квадратную матрицу третьего
порядка:
Определителем матрицы третьего порядка
называется число, которое вычисляется
по формуле:
Пример. Вычислить определитель
третьего порядка:
Решение.
Чтобы получить правило для вычисления
определителя любого порядка, введем
понятие минора и алгебраического дополнения элемента
квадратной
матрицы
.
Минором
элемента матрицы n-го порядка
называется определитель (n-1)-го
порядка, полученный из матрицы А
вычеркиванием i-й строки
и j-го столбца.
Алгебраическим дополнением элемента матрицы n-го порядка
называется его минор, взятый со знаком
,
то есть
Пример. Дана матрица третьего
порядка:
Найти миноры и алгебраические дополнения
Решение. Вычеркивая первую строку
и первый столбец, получим минор Вычеркивая первую строку и второй
столбец, найдем минор
Тогда
Ответ.
За правило вычисления определителя
n-го порядка примем
утверждение следующей теоремы.
Теорема Лапласа. Определитель
квадратной матрицы равен сумме
произведений элементов любой строки
(столбца) на соответствующие алгебраические
дополнения.
,
где i=1,2,…,n.
Эта формула называется разложением
определителя по элементам i-й
строки. Аналогично имеет место разложение
по элементам j-го столбца:
где j=1,2,…,n.
Убедимся в справедливости теоремы на
примере определителя третьего порядка,
разложив его по элементам первой строки.
Полученный ответ совпадает с определением
Пример. Вычислить определитель
квадратной матрицы третьего порядка:
Решение. Разложим определитель
по элементам первой строки
Решение. Раскроем определитель
данной матрицы по элементам первого
столбца
Заметим, что определитель равен
произведению элементов, стоящих на
главной диагонали. И вообще, если
квадратная матрица имеет под главной
диагональю или над ней элементы равные
нулю, то ее определитель равен произведению
чисел главной диагонали
Рассмотрим свойства определителей,
которые можно доказать с помощью теоремы
Лапласа.
При транспонировании матрицы ее
определитель не меняется. Из этого
свойства следует, что все свойства,
сформулированные относительно строк,
справедливы и относительно столбцов.
Если все элементы какой-либо строки
имеют общий множитель, то его можно
вынести за знак определителя:
При перестановке двух строк матрицы
ее определитель меняет знак на
противоположный.
Определитель, имеющий нулевую строку,
равен нулю.
Если определитель имеет две одинаковые
строки, то он равен нулю.
Если определитель имеет две
пропорциональные строки, то он равен
нулю.
7. Если все элементы какой-либо строки
представляют сумму двух слагаемых, то
определитель можно представить как
сумму двух определителей: у первого в
соответствующей строке стоят первые
слагаемые, а у второго – вторые, остальные
элементы те же, что и у данного определителя:
8. Определитель матрицы не изменится,
если к элементам какой-либо строки
матрицы прибавить элементы другой
строки, умноженные на одно и то же число.
9. Сумма произведений элементов
какой-либо строки матрицы на алгебраические
дополнения элементов другой строки
этой матрицы равна нулю.
10. Определитель произведения двух
квадратных матриц равен произведению
их определителей.,
где А,В- матрицы n-го
порядка. То есть если даже АВ,
Приведенные свойства определителей
используются при их вычислении.
Пример. Вычислить определитель
матрицы
Решение. Выполним преобразования,
которые по свойству 8 не изменят величины
определителя: первую строку прибавим
ко второй, и, умноженную на 2, вычтем из
последней строки:
Определитель третьего порядка можно
вычислить по определению или продолжить
применение свойства 8: третью строку,
умноженную на 4, вычитаем из первой
строки, умноженную на 3 вычтем из второй,
получим
Примеры для самостоятельной работы
Вычислить определители:
1) 2)
3)
Ответы: 1)24; 2)0; 3)10.
Determinant of a 3×3 matrix and example
目次
—
$3\times 3$ determinant
—
Example
—
Calculator
—
Правило Сарруса
$3 \times 3$ определитель
Найдите определитель матрицы 3×3,
,
с помощью расширения кофактора.
Доказательство Кофакторное разложение $A$ по первому столбцу равно
Вычисляя определитель 2×2 в каждом члене,
Мы получаем
Примеры
Найдите определители следующих матриц.
Пример ответа По формуле определителя $3 \times 3$,
Калькулятор Введите матрицу 3×3 и нажмите кнопку «Выполнить». Затем выводится определитель.
1
2
3
1
2
3
$|А|$
«=»
Правление Сарруса
Определитель $3 \times 3$ равен
немного сложнее, чем определитель $2 \times 2$,
так что есть визуальная формула для запоминания. Проведем пять линий из левого верхнего угла в правый нижний на матрице $3\times 3$.
Произведение всех элементов, проходящих через 3-ю прямую, равно $A_{11}A_{22}A_{33}$. Произведение всех элементов через 2-ю и 5-ю строки равно $A_{12}A_{23}A_{31}$. Произведение всех элементов через 1-ю и 4-ю строки равно $A_{13}A_{21}A_{32}$. Сумма произведений выше равна
$$
\тег{4. 1}
$$
Далее нарисуем пять линий
из правого верхнего угла в левый нижний угол матрицы.
Произведение всех элементов, проходящих через третью прямую, равно $A_{13}A_{22}A_{31}$. Произведение всех элементов через 2-ю и 5-ю строки равно $A_{12}A_{21}A_{33}$. Произведение всех элементов через 1-ю и 4-ю строки равно $А_{11}А_{23}А_{32}$ Умножение вышеуказанных продуктов на $-1$ и их сложение дает
Сумма произведений выше, умноженная на $-1$, равна
$$
\тег{4.2}
$$
Добавляя $(4.1)$ и $(4.2)$,
у нас есть
, который равен определителю матрицы $3 \times 3$. Таким образом,
определитель матрицы $3 \times 3$
получается путем сложения членов, полученных путем рисования линий из левого верхнего угла в правый нижний,
и вычитание членов, полученных путем рисования линий справа вверху слева внизу.
Эта визуальная формула называется правилом Сарруса.
Расширения кофактора
Цели
Научитесь распознавать, какие методы лучше всего подходят для вычисления определителя данной матрицы.
Рецептов: определитель матрицы 3×3, вычислите определитель, используя разложения кофакторов.
Словарные слова: минор , кофактор .
В этом разделе мы даем рекурсивную формулу для определителя матрицы, называемую разложением кофактора . Формула является рекурсивной в том смысле, что мы будем вычислять определитель матрицы n×n , предполагая , что мы уже знаем, как вычислить определитель матрицы (n−1)×(n−1).
В конце есть дополнительный подраздел о правиле Крамера и формуле кофактора для обратной матрицы.
Рекурсивная формула должна иметь начальную точку. Для расширений кофакторов отправной точкой является случай матриц 1 × 1. Из определения определителя прямо следует, что
detAaB=а.
Чтобы описать разложения кофакторов, нам нужно ввести некоторые обозначения.
Определение
Пусть A — матрица размера n × n.
(i, j) минор, , обозначаемый Aij, представляет собой матрицу (n−1)×(n−1), полученную из A удалением i-й строки и j-го столбца.
Кофактор (i,j) Cij определяется в терминах минора как
Cij=(−1)i+jdet(Aij).
Обратите внимание, что знаки кофакторов расположены в шахматном порядке. А именно, (−1)i+j изображено в этой матрице:
GKI+-+—+-++-+—+-+HLJ.
Пример
Кофакторы Cij матрицы n×n являются определителями (n−1)×(n−1) подматриц. Следовательно, следующая теорема фактически представляет собой рекурсивную процедуру вычисления определителя.
Теорема (расширение кофактора)
Пусть A — матрица размера n × n с элементами aij.
Для любого i=1,2,…,n имеем
det(A)=nMj=1aijCij=ai1Ci1+ai2Ci2+···+ainCin.
Это называется расширением кофактора вдоль i-й строки.
Для любого j=1,2,. ..,n имеем
det(A)=nMi=1aijCij=a1jC1j+a2jC2j+···+anjCnj.
Это называется расширением кофактора по j-му столбцу.
Доказательство
Сначала мы докажем, что разложение кофакторов по первому столбцу вычисляет определитель. Определить функцию d:{n×nmatrices}→R на
d(A)=nMi=1(−1)i+1ai1det(Ai1).
Мы хотим показать, что d(A)=det(A). Вместо того чтобы показывать, что d удовлетворяет четырем определяющим свойствам определителя в разделе 4.1, мы докажем, что он удовлетворяет трем альтернативным определяющим свойствам в разделе 4.1, эквивалентность которых была показана.
Покажем, что d полилинейно по строкам матрицы A. Пусть A — матрица со строками v1,v2,…,vi−1,v+w,vi+1,…,vn:
А=Ea11a12a13b1+c1b2+c2b3+c3a31a32a33F.
Здесь мы пусть bi и ci будут элементами v и w соответственно. Пусть B и C — матрицы со строками v1,v2,…,vi−1,v,vi+1,…,vn и v1,v2,…,vi−1,w,vi+1 ,…,вн соответственно:
Мы хотим показать d(A)=d(B)+d(C). При iAB=i (iA,1)-кофактор A представляет собой сумму (iA,1)-кофакторов B и C в силу полилинейности определителей матриц (n−1)×(n−1) :
по желанию. Это показывает, что d(A) удовлетворяет первому определяющему свойству в строках A.
Нам еще нужно показать, что d(A) удовлетворяет второму определяющему свойству в строках матрицы A. Пусть B — матрица, полученная масштабированием i-й строки матрицы A с коэффициентом c:
по желанию. Это завершает доказательство полилинейности d(A) по строкам матрицы A.
Теперь покажем, что d(A)=0, если A имеет две одинаковые строки. Предположим, что строки i1,i2 матрицы A идентичны, причем i1 A=GKIa11a12a13a14a21a22a23a24a31a32a33a34a11a12a13a14HLJ. Если iB=i1,i2, то (i,1)-сомножитель матрицы A равен нулю, поскольку Ai1 — матрица (n−1)×(n−1) с одинаковыми строками:
Два оставшихся кофактора компенсируются, поэтому d(A)=0, как и требовалось.
Осталось показать, что d(In)=1. Первый — единственный ненулевой член в кофакторном разложении тождества:
d(In)=1·(-1)1+1det(In-1)=1.
Это доказывает, что det(A)=d(A), т. е. разложение на кофакторы по первому столбцу вычисляет определитель.
Теперь мы покажем, что разложение кофактора по j-му столбцу также вычисляет определитель. Выполняя j-1 перестановку столбцов, можно переместить j-й столбец матрицы в первый столбец, сохраняя порядок в других столбцах. Например, здесь мы перемещаем третий столбец в первый, используя два обмена столбцами:
Let B be the matrix obtained by moving the jth column of A to the first column in this way. Тогда (i,j)-минор Aij равен (i,1)-минору Bi1, так как удаление i-го столбца A равносильно удалению первого столбца B. По построению (i,j)-элемент aij матрицы A равен (i,1)-элементу bi1 матрицы B. Поскольку мы знаем, что можем вычислить определители, разложив по первому столбцу, имеем
Это доказывает, что разложение кофактора по i-му столбцу вычисляет определитель A.
В соответствии со свойством транспонирования в разделе 4.1 расширение кофактора по i-й строке A совпадает с расширением кофактора по i-му столбцу AT. Опять же, благодаря свойству транспонирования, мы имеем det(A)=det(AT), поэтому разложение кофакторов вдоль строки также вычисляет определитель.
Обратите внимание, что теорема на самом деле дает 2n различных формул для определителя: по одной для каждой строки и по одной для каждого столбца. Например, формула разложения кофактора по первому столбцу равна
Все второстепенные матрицы являются матрицами 1×1. Поскольку мы видели, что определитель матрицы 1 × 1 — это просто число внутри нее, следовательно, кофакторы равны
Формула определителя матрицы 3×3 выглядит слишком сложной, чтобы ее сразу запомнить. К счастью, есть следующий мнемонический прием.
Рецепт: вычисление определителя матрицы 3×3
Чтобы вычислить определитель матрицы 3×3, сначала нарисуйте матрицу большего размера, в которой первые два столбца повторяются справа. Затем сложите произведения нисходящих диагоналей и вычтите произведения восходящих диагоналей:
В качестве альтернативы нет необходимости повторять первые два столбца, если вы позволяете своим диагоналям «оборачивать» стороны матрицы, как в Pac-Man или Asteroids.
Пример
Расширения кофакторов наиболее полезны при вычислении определителя матрицы, которая имеет строку или столбец с несколькими нулевыми элементами. В самом деле, если (i,j)-элемент A равен нулю, то нет причин вычислять (i,j)-кофактор. В следующем примере мы вычисляем определитель матрицы с двумя нулями в четвертом столбце путем расширения кофакторов по четвертому столбцу.
Пример
Расширения кофакторов также очень полезны при вычислении определителя матрицы с неизвестными элементами. Действительно, в этом случае выполнять редукцию строк неудобно, потому что нельзя быть уверенным, является ли запись, содержащая неизвестное, опорной или нет.
Пример
Часто наиболее эффективно использовать комбинацию нескольких методов при вычислении определителя матрицы. Действительно, при разложении кофакторов на матрице можно вычислить определители кофакторов любым удобным способом. Или можно выполнить операции со строками и столбцами, чтобы очистить некоторые элементы матрицы перед расширением кофакторов, как в предыдущем примере.
Резюме: методы вычисления определителей
У нас есть несколько способов вычисления определителей:
Специальные формулы для матриц 2×2 и 3×3.
Обычно это лучший способ вычислить определитель маленькой матрицы, за исключением матрицы 3×3 с несколькими нулевыми элементами.
Расширение кофактора.
Обычно это наиболее эффективно, когда есть строка или столбец с несколькими нулевыми элементами или если в матрице есть неизвестные элементы.
Операции со строками и столбцами.
Как правило, это самый быстрый результат при представлении большой матрицы, в которой нет ни строки, ни столбца с большим количеством нулей.
Любая комбинация вышеперечисленного.
Расширение кофактора является рекурсивным, но определители миноров можно вычислить любым удобным способом. Или вы можете выполнять операции со строками и столбцами, чтобы очистить некоторые элементы матрицы перед расширением кофакторов.
Помните, все методы вычисления определителя дают одно и то же число.
Напомним из этого предложения в разделе 3.5, что можно вычислить определитель матрицы 2 × 2, используя правило
A=Nd-b-caO=⇒A-1=1det(A)Nd-b-caO.
В этом примере мы вычислили кофакторы матрицы 2×2; используя C11=d,C12=-c,C21=-b,C22=a, мы можем переписать приведенную выше формулу как
A-1=1det(A)NC11C21C12C22O.
Оказывается, эта формула обобщается на матрицы размера n×n.
Теорема
Пусть A — обратимая матрица размера n × n с кофакторами Cij. Затем
Обратите внимание, что (i,j) кофактор Cij входит в запись (j,i) сопряженной матрицы, а не в запись (i,j): сопряженная матрица представляет собой транспонирование матрицы кофакторов.
Пример
Из предыдущего примера видно, что (4.2.1) — очень неэффективный способ вычисления обратной матрицы по сравнению с увеличением на единичную матрицу и сокращением строк, как в этом подразделе раздела 3.5. Тем не менее, у него есть свое применение.
Если матрица имеет неизвестные элементы, то трудно вычислить ее обратную с помощью редукции строк по той же причине, по которой таким способом трудно вычислить определитель: нельзя быть уверенным, является ли элемент, содержащий неизвестное, опорным или нет.
Эта формула полезна для теоретических целей. Обратите внимание, что единственные знаменатели в (4.2.1) появляются при делении на определитель: вычисление кофакторов включает только умножение и сложение, а не деление. Это означает, например, что если определитель очень мал, то любая ошибка измерения в элементах матрицы сильно увеличивается при вычислении обратной. Таким образом, (4.2.1) полезен при анализе ошибок.
В доказательстве теоремы используется интересный трюк под названием Правило Крамера , которое дает формулу для элементов решения обратимого матричного уравнения.
Правило Крамера
Пусть x=(x1,x2,…,xn) — решение уравнения Ax=b, где A — обратимая матрица размера n × n, а b — вектор в Rn. Пусть Ai — матрица, полученная из A заменой i-го столбца на b. Затем
xi=det(Ai)det(A).
Доказательство
Сначала предположим, что A — единичная матрица, так что x=b. Тогда матрица Ai выглядит так:
ГКИ10б1001б2000б3000б41ХЛДЖ.
Раскладывая сомножители по i-й строке, мы видим, что det(Ai)=bi, поэтому в данном случае
xi=bi=det(Ai)=det(Ai)det(A).
Теперь пусть A — общая матрица размера n × n. Один из способов решить Ax=b — сократить расширенную матрицу (A|b) по строке; результат (In|x). В случае, который мы рассмотрели выше, достаточно проверить, что величина det(Ai)/det(A) не меняется, когда мы выполняем операцию со строкой для (A|b), поскольку det(Ai)/det(A) =xi, когда A=In.
Doing a row replacement on (A|b) does the same row replacement on A and on Ai:Ea11a12a13b1a21a22a23b2a31a32a33b3FR2=R2−2R3−−−−−−→Ea11a12a13b1a21−2a31a22−2a32a23−2a33b2−2b3a31a32a33b3FEa11a12a13a21a22a23a31a32a33FR2=R2−2R3−− ——→Ea11a12a13a21-2a31a22-2a32a23-2a33a31a32a33FEa11b1a13a21b2a23a31b3a33FR2=R2-2R3——→Ea11b1a13a21-2a31b2-2b3a23-2a33a31b3a.3a.
В частности, det(A) и det(Ai) не изменились, поэтому det(A)/det(Ai) не изменились.
Масштабирование строки (A|b) с коэффициентом c приводит к масштабированию одной и той же строки A и Ai с одинаковым коэффициентом:
Ea11a12a13b1a21a22a23b2a31a32a33b3FR2=cR2−−−−→Ea11a12a13b1ca21ca22ca23cb2a31a32a33b3FEa11a12a13a21a22a23a31a32a33FR2=cR2−−−−→Ea11a12a13ca21ca22ca23a31a32a33FEa11b1a13a21b2a23a31b3a33FR2=cR2−−−−→Ea11b1a13ca21cb2ca23a31b3a33F.
Тригонометрическая функция Cos() в Python: примеры применения
В этом уроке мы собираемся обсудить тригонометрическую функцию косинуса(cos) в Python. Мы поговорим о модулях, которые мы можем использовать для реализации функции cos в нашей программе Python. Мы также узнаем о построении графиков с помощью функции cos в программе. Итак, давайте начнем с рассмотрения модулей, которые мы можем импортировать в программу для использования функции cos.
Модули Python для функции cos
В Python у нас есть математический модуль, который мы можем использовать для импорта и реализации функции cos, а также других важных математических операций в программе.
Помимо математического модуля, мы также можем использовать модуль numpy Python для реализации функции cos в программе. Мы изучим использование обоих модулей, т. е. модуля math и модуля numpy.
Метод 1: функция cos() в модуле math
Математический модуль Python содержит ряд важных математических значений и операций, и функция cos() является одной из них. Мы можем использовать функцию cos() модуля math для реализации тригонометрического значения cos в программе.
Функция math.cos() возвращает значение тригонометрического косинуса для аргумента, который мы указываем внутри функции, т. е. значение степени в косинусе. Значение, которое мы даем в качестве аргумента функции, должно быть в радианах.
Ниже приведен синтаксис использования функции math.cos() в программе Python:
math.cos(a)
Параметры: Здесь параметр a = значение в радианах.
Возвращаемое значение: функция math.cos() возвращает значение косинуса для аргумента ‘a’ в радианах, которое мы указали внутри функции.
Давайте разберемся с использованием функции cos() модуля math в Python с помощью следующего примера программы:
# Import math module
import math
# Define an input radian value
x = math.pi / 12
# Printing cosine value for respective input value
print("The cosine value of pi / 12 value as given is : ", end ="")
print(math. cos(x))
Выход:
The cosine value of pi / 12 value as given is: 0.9659258262890683
Метод 2: функция cos() в модуле Numpy
Помимо математического модуля, мы также можем использовать модуль numpy для реализации значения тригонометрического косинуса в программе. Для этого нам предоставляется функция cos() внутри модуля numpy, которая дает нам математическое значение косинуса на выходе.
Как и функция math.cos(), при использовании функции cos() модуля numpy мы должны указать значение аргумента в радианах внутри функции.
Ниже приведен синтаксис использования функции numpy.cos() в программе Python:
numpy.cos(a)
Параметры: мы можем указать ‘a’ в качестве следующих типов параметров внутри функции numpy.cos():
В функции можно указать аргумент с одним значением в радианах.
Мы также можем предоставить массив, содержащий несколько значений в радианах, в качестве аргумента функции.
Тип возвращаемого значения: функция numpy.cos() возвращает значения косинуса заданного числа.
Давайте разберемся с использованием функции cos() модуля numpy в Python с помощью следующего примера программы:
# importing numpy module as jtp in program
import numpy as jtp
# defining multiple input values in a single array
ValArray = [0, jtp.pi / 4, jtp.pi / 7, jtp.pi/9, jtp.pi/12, jtp.pi/5]
# printing input array in output
print("Values given in the input array: \n", ValArray)
# using cos() function to get cosine values
CosArray = jtp.cos(ValArray)
# printing cos values in output
print("\nRespective Cosine values for input array values: \n", CosArray)
Выход:
Values given in the input array:
[0, 0.7853981633974483, 0.4487989505128276, 0.3490658503988659, 0.2617993877991494, 0.6283185307179586]
Respective Cosine values for input array values:
[1. 0.70710678 0.90096887 0.93969262 0.96592583 0.80901699]
Построение графика значений косинуса
До сих пор мы изучали использование функции cos() для модулей numpy и math внутри программы Python. Теперь мы будем использовать модули numpy и math, а также функцию cos() для построения графика значений косинуса. Мы можем сделать это графическое представление двумя способами:
Прямой импорт и реализация функции cos() и модуля numpy & math.
Итерация по функции cos() с модулем numpy и math.
Давайте разберемся в реализации обоих методов, используя их в программе Python и построив графики с ними на выходе.
Пример 1: Прямой импорт и реализация функции cos() и модуля numpy & math.
# importing numpy module as jtp
import numpy as jtp
# importing matplotlib module as mlt
import matplotlib.pyplot as mlt
# Defining an array containing radian values
RadValArray = jtp. linspace(-(2*jtp.pi), 2*jtp.pi, 20)
# cosine values for respective array value
CosValArray = jtp.cos(RadValArray)
# printing values in output
print("Radian values in the array: ", RadValArray)
print("\nRespective cos values of array: ", CosValArray)
# using plot() function with variables
mlt.plot(RadValArray, CosValArray, color = 'blue', marker = "*")
mlt.title("Graphical representation of cos function")
mlt.xlabel("X-axis")
mlt.ylabel("Y-axis")
# plotting graph in output
mlt.show()
Пример 2: Итерация по функции cos() с модулем numpy и math.
# importing math module
import math
# importing numpy module as jtp
import numpy as jtp
# importing matplotlib module as mlt
import matplotlib.pyplot as mlt
# Defining an array containing radian values
RadValArray = jtp.linspace(-(2*jtp.pi), 2*jtp.pi, 20)
# Empty array for cosine values
CosValArray = []
#Iterating over the cos values array
for j in range(len(RadValArray)):
CosValArray.append(math.cos(RadValArray[j]))
j += 1
# printing respective values in output
print("Radian values in the array: ", RadValArray)
print("\nRespective cos values of array: ", CosValArray)
# using plot() function with variables
mlt. plot(RadValArray, CosValArray, color = 'orange', marker = "+")
mlt.title("Graphical representation of cos function")
mlt.xlabel("X-axis")
mlt.ylabel("Y-axis")
# plotting graph in output
mlt.show()
Изучаю Python вместе с вами, читаю, собираю и записываю информацию опытных программистов.
Еще для изучения:
Что означает в математике запись у = f(x) — Гипермаркет знаний. Урок «Как построить график функции у = f(kx), если известен график функции y = f(x)»
Что означает в математике запись у = f(x) — Гипермаркет знаний. Урок «Как построить график функции у = f(kx), если известен график функции y = f(x)»
Соблюдение Вашей конфиденциальности важно для нас. По этой причине, мы разработали Политику Конфиденциальности, которая описывает, как мы используем и храним Вашу информацию. Пожалуйста, ознакомьтесь с нашими правилами соблюдения конфиденциальности и сообщите нам, если у вас возникнут какие-либо вопросы.
Сбор и использование персональной информации
Под персональной информацией понимаются данные, которые могут быть использованы для идентификации определенного лица либо связи с ним.
От вас может быть запрошено предоставление вашей персональной информации в любой момент, когда вы связываетесь с нами.
Ниже приведены некоторые примеры типов персональной информации, которую мы можем собирать, и как мы можем использовать такую информацию.
Какую персональную информацию мы собираем:
Когда вы оставляете заявку на сайте, мы можем собирать различную информацию, включая ваши имя, номер телефона, адрес электронной почты и т.д.
Как мы используем вашу персональную информацию:
Собираемая нами персональная информация позволяет нам связываться с вами и сообщать об уникальных предложениях, акциях и других мероприятиях и ближайших событиях.
Время от времени, мы можем использовать вашу персональную информацию для отправки важных уведомлений и сообщений.
Мы также можем использовать персональную информацию для внутренних целей, таких как проведения аудита, анализа данных и различных исследований в целях улучшения услуг предоставляемых нами и предоставления Вам рекомендаций относительно наших услуг.
Если вы принимаете участие в розыгрыше призов, конкурсе или сходном стимулирующем мероприятии, мы можем использовать предоставляемую вами информацию для управления такими программами.
Раскрытие информации третьим лицам
Мы не раскрываем полученную от Вас информацию третьим лицам.
Исключения:
В случае если необходимо — в соответствии с законом, судебным порядком, в судебном разбирательстве, и/или на основании публичных запросов или запросов от государственных органов на территории РФ — раскрыть вашу персональную информацию. Мы также можем раскрывать информацию о вас если мы определим, что такое раскрытие необходимо или уместно в целях безопасности, поддержания правопорядка, или иных общественно важных случаях.
В случае реорганизации, слияния или продажи мы можем передать собираемую нами персональную информацию соответствующему третьему лицу – правопреемнику.
Защита персональной информации
Мы предпринимаем меры предосторожности — включая административные, технические и физические — для защиты вашей персональной информации от утраты, кражи, и недобросовестного использования, а также от несанкционированного доступа, раскрытия, изменения и уничтожения.
Соблюдение вашей конфиденциальности на уровне компании
Для того чтобы убедиться, что ваша персональная информация находится в безопасности, мы доводим нормы соблюдения конфиденциальности и безопасности до наших сотрудников, и строго следим за исполнением мер соблюдения конфиденциальности.
Определение. Пусть функция \(y = f(x) \) определена в некотором интервале, содержащем внутри себя точку \(x_0 \).
Дадим аргументу приращение \(\Delta x \) такое, чтобы не выйти из этого интервала. Найдем соответствующее приращение функции
\(\Delta y \) (при переходе от точки \(x_0 \) к точке \(x_0 + \Delta x \)) и составим отношение
\(\frac{\Delta y}{\Delta x} \). Если существует предел этого отношения при \(\Delta x \rightarrow 0 \), то
указанный предел называют производной функции \(y=f(x) \) в точке \(x_0 \) и обозначают \(f»(x_0) \).
Для обозначения производной часто используют символ y». Отметим, что y» = f(x) — это новая функция, но, естественно, связанная с функцией y = f(x), определенная во всех точках x, в которых
существует указанный выше предел. Эту функцию называют так: производная функции у = f(x) .
Геометрический смысл производной состоит в следующем. Если к графику функции у = f(x) в точке с абсциссой х=a можно
провести касательную, непараллельную оси y, то f(a) выражает угловой коэффициент касательной: \(k = f»(a) \)
Поскольку \(k = tg(a) \), то верно равенство \(f»(a) = tg(a) \) .
А теперь истолкуем определение производной с точки зрения приближенных равенств. Пусть функция \(y = f(x) \) имеет
производную в конкретной точке \(x \): $$ \lim_{\Delta x \to 0} \frac{\Delta y}{\Delta x} = f»(x) $$ Это означает, что около точки х выполняется приближенное равенство \(\frac{\Delta y}{\Delta x} \approx f»(x) \), т.е.
\(\Delta y \approx f»(x) \cdot \Delta x \).
Содержательный смысл полученного приближенного равенства заключается в следующем: приращение функции «почти пропорционально»
приращению аргумента, причем коэффициентом пропорциональности является значение производной в заданной точке х. 2 \) справедливо приближенное равенство \(\Delta y \approx 2x \cdot \Delta x \).
Если внимательно проанализировать определение производной, то мы обнаружим, что в нем заложен алгоритм ее нахождения.
Сформулируем его.
Как найти производную функции у = f(x) ?
1. Зафиксировать значение \(x \), найти \(f(x) \) 2. Дать аргументу \(x \) приращение \(\Delta x \), перейти в новую точку \(x+ \Delta x \), найти \(f(x+ \Delta x) \) 3. Найти приращение функции: \(\Delta y = f(x + \Delta x) — f(x) \) 4. Составить отношение \(\frac{\Delta y}{\Delta x} \) 5. Вычислить $$ \lim_{\Delta x \to 0} \frac{\Delta y}{\Delta x} $$ Этот предел и есть производная функции в точке x.
Если функция у = f(x) имеет производную в точке х, то ее называют дифференцируемой в точке х. Процедуру нахождения производной
функции у = f(x) называют дифференцированием функции у = f(x).
Обсудим такой вопрос: как связаны между собой непрерывность и дифференцируемость функции в точке.
Пусть функция у = f(x) дифференцируема в точке х. Тогда к графику функции в точке М(х; f(x)) можно провести касательную,
причем, напомним, угловой коэффициент касательной равен f»(x). Такой график не может «разрываться» в точке М, т. е. функция
обязана быть непрерывной в точке х.
Это были рассуждения «на пальцах». Приведем более строгое рассуждение. Если функция у = f(x) дифференцируема в точке х, то
выполняется приближенное равенство \(\Delta y \approx f»(x) \cdot \Delta x \). Если в этом равенстве \(\Delta x \) устремить к
нулю, то и \(\Delta y \) будет стремиться к нулю, а это и есть условие непрерывности функции в точке.
Итак, если функция дифференцируема в точке х, то она и непрерывна в этой точке .
Обратное утверждение неверно. Например: функция у = |х| непрерывна везде, в частности в точке х = 0, но касательная к графику
функции в «точке стыка» (0; 0) не существует. Если в некоторой точке к графику функции нельзя провести касательную, то в этой
точке не существует производная.
Еще один пример. Функция \(y=\sqrt{x} \) непрерывна на всей числовой прямой, в том числе в точке х = 0.
И касательная к графику функции существует в любой точке, в том числе в точке х = 0. Но в этой точке касательная совпадает с осью у,
т. е. перпендикулярна оси абсцисс, ее уравнение имеет вид х = 0. Углового коэффициента у такой прямой нет, значит, не существует и
\(f»(0) \)
Итак, мы познакомились с новым свойством функции — дифференцируемостью. А как по графику функции можно сделать вывод о ее
дифференцируемости?
Ответ фактически получен выше. Если в некоторой точке к графику функции можно провести касательную, не перпендикулярную оси
абсцисс, то в этой точке функция дифференцируема. Если в некоторой точке касательная к графику функции не существует или она
перпендикулярна оси абсцисс, то в этой точке функция не дифференцируема.
Правила дифференцирования
Операция нахождения производной называется дифференцированием .
При выполнении этой операции часто приходится работать с частными, суммами, произведениями функций, а также с «функциями функций»,
то есть сложными функциями. 2} $$
Если задано множество чисел X и указан способ f , по которому для каждого значения х ЄX ставится в соответствие только одно число у . Тогда считается заданной функция y = f (х ), у которой область определения X (обычно обозначают D (f ) = X ). Множество Y всех значений у , для которых есть как минимум одно значение х ЄX , такое, что y = f (х ), такое множество называют множеством значений функции f (чаще всего обозначают E (f )= Y ).
Или зависимость одной переменной у от другой х , при которой каждому значению переменной х из определенного множества D соответствует единственное значение переменной у , называется функцией .
Функциональную зависимость переменной у от х часто подчеркивают записью у(х), которую читают игрек от икс.
Область определения функции у (х ), т. е. множество значений ее аргумента х , обозначают символом D (y ), который читают дэ от игрек.
Область значений функции у (х ), т. е. множество значений, которые принимает функция у, обозначают символом Е (у ), который читают е от игрек.
Основными способами задания функции являются:
а) аналитический (с помощью формулы y = f (х )). К этому способу можно отнести и случаи, когда функция задается системой уравнений. Если функция задана формулой, то область ее определения составляют все те значения аргумента, при которых выражение, записанное в правой части формулы, имеет значения.
б) табличный (с помощью таблицы соответствующих значений х и у ). Таким способом часто задается температурный режим или курсы валют, но этот способ не такой наглядный, как следующий;
в) графический (с помощью графика). Это один из самых наглядных способов задания функции, поскольку по графику сразу «читаются» изменения. Если функция у (х ) задана графиком, то область ее определения D (y ) есть проекция графика на ось абсцисс, а область значений Е (у ) — проекция графика на ось ординат (смотри рисунок).
г) словестный . Этот способ часто применяется в задачах, а точнее в описании их условия. Обычно этот способ заменяют одним из приведенных выше.
Функции y = f (х ), x ЄX , и y = g (х ), x ЄX , называются тождественно равными на подмножестве М СX , если для каждого x 0 ЄМ справедливо равенство f (х 0) = g (х 0).
График функции y = f (х ) можно представить, как множество таких точек (х ; f (х )) на координатной плоскости, где х — произвольная переменная, из D (f ). Если f (х 0) = 0, где х 0 то точка с координатами (x 0 ; 0) — это точка, в которой график функции y = f (х ) пересекается с осью Оx . Если 0ЄD (f ), то точка (0; f (0)) — это точка, в которой график функции у = f (x ) пересекается с осью Оу .
Число х 0 из D (f ) функции y = f (х ) это нуль функции, тогда, когда f (х 0) = 0.
Промежуток М СD (f ) это промежуток знакопостоянства функции y = f (х ), если либо для произвольного x ЄМ верно f (х ) > 0, либо для произвольного х ЄМ верно f (х )
Есть приборы , которые вырисовывают графики зависимостей между величинами. Это барографы — приборы для фиксации зависимости атмосферного давления от времени, термографы — приборы для фиксации зависимости температуры от времени, кардиографы — приборы для графической регистрации деятельности сердца. У термографа есть барабан, он равномерно вращается. Бумаги, намотанной на барабан, касается самописец, который в зависимости от температуры поднимается и опускается и вырисовывает на бумаге определенную линию.
От представления функции формулой можно перейти к ее представлению таблицей и графиком.
При изучении математики очень важно понимать, что такое функция, ее области определения и значения. С помощью исследования функций на экстремум можно решить многие задачи по алгебре. Даже задачи по геометрии иногда сводятся к рассмотрению уравнений геометрических фигур на плоскости.
Описание видеоурока
Функцией называется зависимость переменной игрек от переменной икс, при которой каждому значению переменной икс соответствует единственное значение переменной игрек.
Икс называется независимой переменной или аргументом. Игрек называется зависимой переменной, значением функции или просто функцией.
Если зависимость переменной игрек от переменной икс является функцией, то коротко записывают так: игрек равно эф от икс. Этим символом обозначают также значение функции, соответствующее значению аргумента икс.
Пусть функция задана формулой игрек равно три икс квадрат минус пять. Тогда можно записать, что эф от икс равно три икс квадрат минус пять. Найдем значения функции эф для значений икс, равных двум и минус пяти. Они будут равны семи и семидесяти.
Заметим, что в записи игрек равно эф от икс вместо эф можно употреблять и другие буквы: же, фи и так далее.
Все значения икс образуют область определения функции. Все значения, которые принимает игрек, образуют область значений функции.
Функция считается заданной, если указана её область определения и правило, согласно которому каждому значению икс поставлено в соответствие единственное значение игрек.
Если функция игрек равно эф от икс задана формулой и ее область определения не указана, то считают, что область определения функции состоит из всех значений переменной икс, при которых выражение эф от икс имеет смысл…
Графиком функции называется множество всех точек координатной плоскости, абсциссы которых равны значениям аргумента, а ординаты — соответствующим значениям функции.
На рисунке изображен график функции игрек равно эф от икс, областью определения которой является отрезок от единицы до пяти. С помощью графика можно найти, например, что функция от числа один равна минус трем, функция от двух равна двум, функция от числа четыре равна минус двум, функция от числа пять равна минус четырем. Наименьшее значение функции равно минус четырем, а наибольшее — двум. При этом любое число от минус четырех до двух, включая эти числа, является значением данной функции. Таким образом, областью значений функции игрек равно эф от икс является отрезок от минус четырех до двух.
Ранее нами уже были изучены некоторые виды функций:
Линейная функция, задаваемая формулой игрек равно ка икс плюс бэ, где ка и бэ — некоторые числа;
Прямая пропорциональность — частный случай линейной функции, она задается формулой игрек равно ка икс, где ка не равно нулю;
Обратная пропорциональность — функция игрек равно ка деленное на икс, где ка не равно нулю.
Графиком функции игрек равно ка икс плюс бэ является прямая. Область определения этой функции — множество всех чисел. Областью значений этой функции при ка не равном нулю является множество всех чисел, а при ка равном нулю ее область значений состоит из одного числа бэ.
График функции игрек равно ка деленное на икс называется гиперболой.
На рисунке изображен график функции игрек равно ка деленное на икс, для ка большего нуля. Областью определения этой функции является множество всех чисел, кроме нуля. Это множество является и областью ее значений…
Функциями описываются многие реальные процессы и закономерности. Например, прямой пропорциональностью является зависимость массы тела от его объема при постоянной плотности; зависимость длины окружности от ее радиуса. Обратной пропорциональностью является зависимость силы тока на участке цепи от сопротивления проводника при постоянном напряжении; зависимость времени, которое затрачивает равномерно движущееся тело на прохождение заданного пути, от скорости движения.
Изучались также функции, заданные формулами игрек равно икс квадрат, игрек равно икс куб, игрек равно корень квадратный из икс.
Рассмотрим функцию, заданную формулой игрек равно модуль икс.
Так как выражение модуль икс имеет смысл при любом икс, то областью определения этой функции является множество всех чисел. По определению модуль икс равен икс, если икс больше либо равен нулю, и минус икс, если икс меньше нуля. Поэтому функцию игрек равно модуль икс можно задать следующей системой.
График рассматриваемой функции в промежутке от нуля до плюс бесконечности, включая ноль, совпадает с графиком функции игрек равно икс, а в промежутке от минус бесконечности до нуля — с графиком функции игрек равно минус икс. График функции игрек равно модуль икс состоит из двух лучей, которые исходят из начала координат и являются биссектрисами первого и второго координатных углов.
1)
Область определения функции и область
значений функции .
Область
определения функции — это множество
всех допустимых действительных значений
аргумента x (переменной x ),
при которых функция y
= f(x) определена. Область
значений функции — это множество всех
действительных значений y ,
которые принимает функция.
В
элементарной математике изучаются
функции только на множестве действительных
чисел.
2)
Нули функции .
Нуль
функции – такое значение аргумента,
при котором значение функции равно
нулю.
3)
Промежутки знакопостоянства функции .
Промежутки
знакопостоянства функции – такие
множества значений аргумента, на которых
значения функции только положительны
или только отрицательны.
4)
Монотонность функции .
Возрастающая
функция (в некотором промежутке) —
функция, у которой большему значению
аргумента из этого промежутка
соответствует большее значение функции.
Убывающая
функция (в некотором промежутке) —
функция, у которой большему значению
аргумента из этого промежутка
соответствует меньшее значение функции.
5)
Четность (нечетность) функции .
Четная
функция — функция, у которой область
определения симметрична относительно
начала координат и для любого х из
области определения выполняется
равенство f(-x)
= f(x) . График четной функции симметричен
относительно оси ординат.
Нечетная
функция — функция, у которой область
определения симметрична относительно
начала координат и для любого х из
области определения справедливо
равенство f(-x)
= — f(x ).
График нечетной функции симметричен
относительно начала координат.
6)
Ограниченная и неограниченная функции .
Функция
называется ограниченной, если существует
такое положительное число M, что |f(x)| ≤
M для всех значений x . Если такого числа
не существует, то функция — неограниченная.
7)
Периодическость функции .
Функция
f(x) — периодическая, если существует
такое отличное от нуля число T, что для
любого x из области определения функции
имеет место: f(x+T) = f(x). Такое наименьшее
число называется периодом функции. Все
тригонометрические функции являются
периодическими. (Тригонометрические
формулы).
19.
Основные элементарные функции, их
свойства и графики. Применение функ-ций
в экономике.
Основные элементарные функции. Их свойства и графики
1. Линейная функция.
Линейной
функцией называется
функция вида
,
где х — переменная, а и b — действительные
числа.
Число а называют угловым коэффициентом прямой,
он равен тангенсу угла наклона этой
прямой к положительному направлению
оси абсцисс. Графиком линейной функции
является прямая линия. Она определяется
двумя точками.
Свойства линейной функции
1. Область определения
— множество всех действительных чисел:
Д(y)=R
2. Множество значений
— множество всех действительных чисел:
Е(у)=R
3. Функция принимает
нулевое значение при
или.
4. Функция возрастает
(убывает) на всей области определения.
5. Линейная функция
непрерывная на всей области определения,
дифференцируемая и
.
2. Квадратичная функция.
Функция
вида
,
где х — переменная, коэффициенты а, b, с
— действительные числа,
называетсяквадратичной.
Мэтуэй | Популярные задачи
1
Найти точное значение
грех(30)
2
Найти точное значение
грех(45)
3
Найти точное значение
грех(30 градусов)
4
Найти точное значение
грех(60 градусов)
5
Найти точное значение
загар (30 градусов)
6
Найти точное значение
угловой синус(-1)
7
Найти точное значение
грех(пи/6)
8
Найти точное значение
cos(pi/4)
9
Найти точное значение
грех(45 градусов)
10
Найти точное значение
грех(пи/3)
11
Найти точное значение
арктан(-1)
12
Найти точное значение
cos(45 градусов)
13
Найти точное значение
cos(30 градусов)
14
Найти точное значение
желтовато-коричневый(60)
15
Найти точное значение
csc(45 градусов)
16
Найти точное значение
загар (60 градусов)
17
Найти точное значение
сек(30 градусов)
18
Найти точное значение
cos(60 градусов)
19
Найти точное значение
cos(150)
20
Найти точное значение
грех(60)
21
Найти точное значение
cos(pi/2)
22
Найти точное значение
загар (45 градусов)
23
Найти точное значение
arctan(- квадратный корень из 3)
24
Найти точное значение
csc(60 градусов)
25
Найти точное значение
сек(45 градусов)
26
Найти точное значение
csc(30 градусов)
27
Найти точное значение
грех(0)
28
Найти точное значение
грех(120)
29
Найти точное значение
соз(90)
30
Преобразовать из радианов в градусы
пи/3
31
Найти точное значение
желтовато-коричневый(30)
32
92
35
Преобразовать из радианов в градусы
пи/6
36
Найти точное значение
детская кроватка(30 градусов)
37
Найти точное значение
арккос(-1)
38
Найти точное значение
арктан(0)
39
Найти точное значение
детская кроватка(60 градусов)
40
Преобразование градусов в радианы
30
41
Преобразовать из радианов в градусы
(2 шт. )/3
42
Найти точное значение
sin((5pi)/3)
43
Найти точное значение
sin((3pi)/4)
44
Найти точное значение
тан(пи/2)
45
Найти точное значение
грех(300)
46
Найти точное значение
соз(30)
47
Найти точное значение
соз(60)
48
Найти точное значение
соз(0)
49
Найти точное значение
соз(135)
50
Найти точное значение
cos((5pi)/3)
51
Найти точное значение
cos(210)
52
Найти точное значение
сек(60 градусов)
53
Найти точное значение
грех(300 градусов)
54
Преобразование градусов в радианы
135
55
Преобразование градусов в радианы
150
56
Преобразовать из радианов в градусы
(5 дюймов)/6
57
Преобразовать из радианов в градусы
(5 дюймов)/3
58
Преобразование градусов в радианы
89 градусов
59
Преобразование градусов в радианы
60
60
Найти точное значение
грех(135 градусов)
61
Найти точное значение
грех(150)
62
Найти точное значение
грех(240 градусов)
63
Найти точное значение
детская кроватка(45 градусов)
64
Преобразовать из радианов в градусы
(5 дюймов)/4
65
Найти точное значение
грех(225)
66
Найти точное значение
грех(240)
67
Найти точное значение
cos(150 градусов)
68
Найти точное значение
желтовато-коричневый(45)
69
Оценить
грех(30 градусов)
70
Найти точное значение
сек(0)
71
Найти точное значение
cos((5pi)/6)
72
Найти точное значение
КСК(30)
73
Найти точное значение
arcsin(( квадратный корень из 2)/2)
74
Найти точное значение
загар((5pi)/3)
75
Найти точное значение
желтовато-коричневый(0)
76
Оценить
грех(60 градусов)
77
Найти точное значение
arctan(-( квадратный корень из 3)/3)
78
Преобразовать из радианов в градусы
(3 пи)/4
79
Найти точное значение
sin((7pi)/4)
80
Найти точное значение
угловой синус(-1/2)
81
Найти точное значение
sin((4pi)/3)
82
Найти точное значение
КСК(45)
83
Упростить
арктан(квадратный корень из 3)
84
Найти точное значение
грех(135)
85
Найти точное значение
грех(105)
86
Найти точное значение
грех(150 градусов)
87
Найти точное значение
sin((2pi)/3)
88
Найти точное значение
загар((2pi)/3)
89
Преобразовать из радианов в градусы
пи/4
90
Найти точное значение
грех(пи/2)
91
Найти точное значение
сек(45)
92
Найти точное значение
cos((5pi)/4)
93
Найти точное значение
cos((7pi)/6)
94
Найти точное значение
угловой синус(0)
95
Найти точное значение
грех(120 градусов)
96
Найти точное значение
желтовато-коричневый ((7pi)/6)
97
Найти точное значение
соз(270)
98
Найти точное значение
sin((7pi)/6)
99
Найти точное значение
arcsin(-( квадратный корень из 2)/2)
100
Преобразование градусов в радианы
88 градусов
калькулятор синусоиды — Googlesuche
AlleBilderShoppingVideosMapsNewsBücher
Suchoptionen
Sinusoid (Sine wave)
Der Sinusoid ist eine sinusförmige Funktion, die aus der Sinusfunktion durch Skalierung von Amplitude und Frequenz sowie Phasenverschiebung gebildet wird. Er bildet die Grundlage der Darstellung im Frequenzbereich. Wikipedia
Калькулятор синусоиды
planetcalc.com › …
Этот калькулятор строит параметрическую синусоиду в диапазоне от 0 до 2\pi. Почему параметрический? Потому что график представлен следующей формулой.
Калькулятор синуса – Sin(x) | Определение | Графики
www.omnicalculator.com › математика › sin
08.12.2022 · С помощью этого калькулятора sin вы можете найти значение синуса в мгновение ока – все, что вам нужно сделать, это ввести угол в градусах или радианах .
Калькулятор функции SIN и график — Программное обеспечение MedCalc
www.medcalc.org › … › Тригонометрические функции
SIN(x) возвращает синус угла x. … Калькулятор. SIN(1-й аргумент). График Функция СИН. Функция: SIN( ). Ось X, ось Y. Минимум: Минимум X
Калькулятор синусоидальной функции — Бесплатный онлайн-калькулятор — Byju’s
byjus. com › Калькуляторы › Математические калькуляторы
Калькулятор синусоидальной функции — это бесплатный онлайн-инструмент, который отображает волновую картину для заданных входных данных. Онлайн-калькулятор синусоидальной функции BYJU …
Графики тригонометрических функций F(π) — Calculator Soup
www.calculatorsoup.com › Тригонометрия
Графики тригонометрических функций для синуса, косинуса, тангенса, котангенса, секанса и косеканса функция π. Используйте онлайн-калькуляторы для тригонометрии.
Синусоидальные графики — Desmos
www.desmos.com › калькулятор
Untitled Graph. Войдите или зарегистрируйтесь. 1. 2. питается от. питаться от. «х» х. «у» у. «а» в квадрате а 2. «а» Верхний индекс, «б» , базовая линия а б.
Функция Grapher and Calculator — Math is Fun
www.mathsisfun.com › data › function-grapher › fu…
Вы можете использовать «a» в своей формуле, а затем использовать ползунок, чтобы изменить значение из «а», чтобы увидеть, как это влияет на график.
Деление треугольника на равные площади параллельными
Определение основных параметров целого числа
Свойства обратных тригонометрических функций
Разделить шар на равные объемы параллельными плоскостями
Взаимосвязь между организмами с различными типами обмена веществ
Аутотрофные и миксотрофные организмы
Рассечение круга прямыми на равные площади
Период нечетной дроби онлайн. Первые полторы тысяч разложений.
Представить дробь, как сумму её множителей
Решение системы из двух однородных диофантовых уравнений
Расчет основных параметров четырехполюсника
Цепочка остатков от деления в кольце целого числа
Система счисления на базе ряда Фибоначчи онлайн
Уравнение пятой степени. Частное решение.
Рассчитать площадь треугольника по трем сторонам онлайн
Общее решение линейного диофантового неоднородного уравнения
Частное решение диофантового уравнения с несколькими неизвестными
Онлайн разложение дробно рациональной функции
Корни характеристического уравнения
Число или выражение
Разрядность ячейки памяти
Система счисления
Прямой код
Обратный код
Дополнительный код
Выделим в отдельный материал расчет обратного и дополнительного кода отрицательных чисел. Изначально был создан калькулятор разных систем счисления, но несмотря на его универсальность, посетители сайта редко его находили.
Не будем загружать Вас информацией что это за кода. Такого материала валом на других ресурсах.
Что делаете Вы?
Вводите целое число ( отрицательное в том числе) или выражение.
Указываете разрядность ячейки памяти
Указываете в какой системе счисления необходимо получить результат.
Что делает бот?
Высчитывает выражение. Если выражение дробное, то отбрасывает дробную часть
Выдает дополнительный и обратный код в той системе счсиления которая была задана.
Удобное и быстрое решение для ускорения работы.
Открыт ли порт на удаленном сервере >>
Поиск по сайту
Русский и английский алфавит в одну строку
Часовая и минутная стрелка онлайн. Угол между ними.
Массовая доля химического вещества онлайн
Декoдировать текст \u0xxx онлайн
Универсальный калькулятор комплексных чисел онлайн
Перемешать буквы в тексте онлайн
Частотный анализ текста онлайн
Поворот точек на произвольный угол онлайн
Обратный и дополнительный код числа онлайн
Площадь многоугольника по координатам онлайн
Остаток числа в степени по модулю
Расчет пропорций и соотношений
Как перевести градусы в минуты и секунды
Расчет процентов онлайн
Поиск объекта по географическим координатам
Растворимость металлов в различных жидкостях
DameWare Mini Control. Настройка.
Время восхода и захода Солнца и Луны для местности
Калькулятор географических координат
Расчет значения функции Эйлера
Перевод числа в код Грея и обратно
Теория графов. Матрица смежности онлайн
Произвольный треугольник по заданным параметрам
НОД двух многочленов. Greatest Common Factor (GCF)
Географические координаты любых городов мира
Площадь пересечения окружностей на плоскости
Онлайн определение эквивалентного сопротивления
Непрерывные, цепные дроби онлайн
Сообщество животных. Кто как называется?
Проекция точки на плоскость онлайн
Калькулятор онлайн расчета количества рабочих дней
Из показательной в алгебраическую. Подробно
Расчет заряда и разряда конденсатора через сопротивление
Система комплексных линейных уравнений
Расчет понижающего конденсатора
Построить ненаправленный граф по матрице
Месторождения золота и его спутники
Определение формулы касательной к окружности
Дата выхода на работу из отпуска, декрета онлайн
Каноническое уравнение гиперболы по двум точкам
Онлайн расчеты
Подписаться письмом
Коды двоичных чисел
Виды кодов отрицательных чисел
Прямой код двоичного числа
Обратный код двоичного числа
Дополнительный код двоичного числа
В принципы работы вычислительных машин заложен принцип двоичного кодирования: все данные представлены в виде закодированных некоторым образом двоичных чисел. коды двоичных чисел необходимы для того, чтобы производить над данными логические и арифметические операции.
В статье «Системы счисления» мы рассматривали только положительные числа. При записи двоичных чисел со знаком
в их формате необходимо предусмотреть два поля: поле, определяющее знак числа, и поле, характеризующее модуль числа.
Под знак числа отводится специальный знаковый бит (двоичный разряд). Остальные разряды определяют
модуль числа. Знаковый разряд приписывается слева от модуля числа, причём знаку «+» соответствует
нулевое значение знакового бита, а знаку «-» — единичное.
В истории развития компьютеров использовались три основных варианта представления
знаковых чисел:
прямой код или знак и величина;
обратный код или код с дополнением до единицы;
дополнительный код или код с дополнением до двух.
Во всех трёх кодах положительные числа выглядят одинаково. Различия в форме записи
отрицательных чисел в обратном и дополнительном кодах касаются только способа представления модуля числа, а способ
кодирования и место расположения знакового бита остаются неизменными.
В системе представления в прямом коде число состоит из кода знака и модуля числа,
причём обе эти части обрабатываются по отдельности.
Примеры прямого кода для правильных дробей:
Примеры прямого кода для целых чисел:
Представление чисел в прямом коде имеет существенный недостаток — формальное
суммирование чисел с различающимися знаками даёт неверный результат. Пример — сложение двух чисел
и .
В прямом коде эти числа имеют вид: и .
Очевидно, что результат должен быть равен -2, что в прямом коде может быть записано как 1.010. В
то же время при непосредственном сложении получаем
,
то есть значение, существенно отличающееся от ожидаемого.
Процедура для корректного сложения чисел в прямом коде всё же существует, но она очень громоздка.
Прямой код имеет ещё один недостаток — нуль имеет два различных представления, а именно
и ,
что математически не имеет смысла.
По причине отмеченных недостатков в вычислительных машинах используется не прямой код, а обратный и дополнительный коды.
В этих системах кодирования чисел место расположения знакового разряда и способ кодирования
остаются теми же, что и в прямом кодировании. Однако знаковый разряд уже не рассматривается как обособленный,
а считается неотъемлемой частью числа аналогично разрядам модуля числа и совместно с ними.
Для отрицательных двоичных чисел процедура получения
обратного кода следующая: в знаковой разряд записывается единица, а в цифровых разрядах прямого кода
единицы заменяются нулями, а нули единицами.
Примеры обратного кода для правильных дробей:
.
Примеры обратного кода для целых чисел:
.
Как нетрудно заметить, положительные числа в прямом и обратном кодах выглядят одинаково.
Хотя обратный код и позволяет решить проблему сложения и вычитания чисел с различными
знаками, он имеет и недостатки. Во-первых, процесс суммирования чисел является двухэтапным, что увеличивает время выполнения
этой операции. Во-вторых, как и в прямом коде, в обратном — два представления нуля:
и .
Дополнительный код отрицательного двоичного числа
формируется по следующему правилу: в цифровых разрядах прямого кода единицы заменить нулями, а нули —
единицами, после чего к младшему разряду прибавить единицу.
Для примера рассмотрим число X, которое в прямом коде имеет вид:
.
Тогда обратный код можно записать как
.
Для получения дополнительного кода прибавим 1 к младшему разряду обратного кода:
.
Примеры дополнительного кода для правильных дробей:
.
Примеры дополнительного кода для целых чисел:
.
Положительные числа в дополнительном коде записываются так же, как и в прямом. При представлении чисел в дополнительном коде есть только одна форма записи нуля: 0.0…00,
причём ноль считается положительным числом, так как его знаковый бит равен 0.
В большинстве вычислительных машин отрицательные числа представлены в дополнительном коде.
Сложение и вычитание чисел в обратном и дополнительном кодах
Вычитание производится как сложение чисел, одно из которых с отрицательным знаком.
При выполнении алгебраического сложения знаковый разряд и цифры модуля рассматриваются как
единое целое и обрабатываются совместно. Перенос из старшего (знакового) разряда в обратном и дополнительном кодах
учитывается по-разному. В случае обратного кода единица переноса из знакового разряда прибавляется
к младшему разряду суммы. При использовании дополнительного кода единица переноса из знакового разряда отбрасывается.
Пример 1. Сложить числа и
При использовании обратного кода получим:
При использовании дополнительного кода получим:
Если знаковый разряд результата равен нулю, это означает, что получено положительное число,
которое выглядит так же, как и в прямом коде. Единица в знаковом разряде означает, что результат отрицательный
и его запись соответствует представлению в том коде, в котором производилась операция.
К началу страницы
Reverse Binary Number — Reverse Bits — Online
Самый простой в мире реверсер двоичных битов для веб-разработчиков и программистов. Просто вставьте свои двоичные числа в форму ниже, нажмите Reverse Bits, и вы получите перевернутые двоичные строки. Нажмите кнопку, обратный двоичный код. Никакой рекламы, ерунды или мусора.
Объявление : Мы только что добавили три новые категории инструментов — Текстовые инструменты, Инструменты для изображений и Математические инструменты. Проверь их!
(отменить)
Хотите инвертировать двоичные числа?
Используйте инструмент «Двоичный инвертор»!
Хотите вращать двоичные числа?
Используйте инструмент Binary Rotator!
Ищете дополнительные инструменты для веб-разработчиков? Попробуйте эти!
Кодировщик URL
Декодер URL
Анализатор URL
Кодировщик HTML
Декодер HTML
Кодировщик Base64
Декодер Base64
HTML Минификатор
JSON Prettifier
JSON Minifier
JSON Escaper
JSON Unescaper
JSON Validator
JS Prettifier
JS Minifier
JS Validator
CSS Prettify
CSS0 Minifier 090 XML Prettify 2 Minifier
Преобразователь XML в JSON
Преобразователь JSON в XML
Преобразователь XML в CSV
Преобразователь CSV в XML
Преобразователь XML в YAML
Преобразователь YAML в XML
Преобразователь YAML в TSV
Преобразователь TSV в YAML
Преобразователь XML в TSV
Преобразователь TSV в XML
Преобразователь XML в текст
Преобразователь JSON в CSV
Преобразователь CSV в JSON 02 Конвертер JSON в TSV
Конвертер TSV в JSON
Конвертер JSON в текст
Конвертер CSV в YAML
Конвертер YAML в CSV
Конвертер TSV в CSV
Конвертер CSV в TSV
90 Text002 Конвертер CSV в столбец
Преобразователь текстовых столбцов в CSV
Преобразование TSV в текстовые столбцы
Преобразование текстовых столбцов в TSV
Преобразование CSV
Преобразование столбцов CSV в строки
Преобразование столбцов CSV в 0 0 строк Средство замены столбцов CSV
Преобразование восьмеричных чисел в шестнадцатеричные
Преобразование десятичных чисел в двоичные
Преобразование десятичных чисел в восьмеричные
Преобразование десятичных чисел в шестнадцатеричные
Преобразование шестнадцатеричных чисел в двоичные 9002x
900 0002 Конвертер шестнадцатеричных чисел в десятичные
Конвертер десятичных чисел в двоично-десятичные
Конвертер двоично-десятичных чисел
Конвертер восьмеричных чисел в двоично-десятичные
Конвертер двоично-десятичных чисел в восьмеричные
Конвертер шестнадцатеричных чисел в двоично-десятичные
Конвертер двоично-десятичных чисел
Двоичное преобразование в серое
Серое в двоичное Преобразование
Восьмеричное преобразование в серое
Серое в восьмеричное Шестнадцатеричный преобразователь
Калькулятор двоичной суммы
Калькулятор двоичного произведения
Калькулятор двоичного побитового И
Калькулятор двоичного побитового И-НЕ
Калькулятор двоичного побитового ИЛИ
Двоичный побитовый калькулятор НЕ-ИЛИ
Двоичный побитовый калькулятор исключающего ИЛИ
Двоичный побитовый калькулятор исключающего ИЛИ
Двоичный побитовый калькулятор НЕ
Двоичный инвертор битов
Двоичный битовый инвертор
Двоичный битовый реверс
9
Двоичный поворотник влево
Двоичный Вращатель битов вправо
Преобразователь числа
Преобразователь римских чисел в десятичные
Преобразователь десятичных чисел в римские
Преобразователь чисел в слова
Преобразователь слов в числа
Округление чисел вверх
Округление чисел вниз
Преобразование UTF8 в Hex
Преобразование Hex в UTF8
Преобразование текста в коды ASCII Двоичный преобразователь
Двоичный Преобразователь текста в текст
Преобразователь текста в восьмеричный
Преобразователь восьмеричного в текст
Преобразователь текста в десятичный
Преобразователь десятичного в текст
Преобразователь текста в шестнадцатеричный
Преобразователь шестнадцатеричного формата в текст
Преобразователь текста в нижний регистр
Преобразователь текста в верхний регистр
Преобразователь текста в случайный регистр
Обрезать текст Lines
Преобразователь пробелов в символы табуляции
Преобразователь символов табуляции в пробелы
Преобразователь пробелов в новые строки
Преобразователь новых строк в пробелы
Средство удаления акцента
Удаление лишних пробелов
Удаление всех пробелов
Удаление знаков препинания
Добавление разделителя тысяч
Удаление обратной косой черты
Добавление обратной косой черты
0 90 90 90 Преобразование текста 003
Замена текста
Реверс текста
Поворот текста
Вращатель текстовых символов влево
Вращатель текстовых символов вправо
Калькулятор длины текста
Алфавитный сортировщик текста
Числовой сортировщик текста
Сортировщик текста по длине
Текст из генератора регулярных выражений
Текст по центру
Текст с выравниванием по правому краю
Текст с левой панелью
Справа 0 текст 290 Текст
9000 Средство форматирования столбцов
Regex Match Extractor
Regex Match Replacer
Email Extractor
URL Extractor
Number Extractor
List Merger
List Zipper
List Intersection
Различие списка
Программа форматирования Printf
Группа текста
Заголовок текста
Конец текста
Извлечение диапазона строк
Сортировщик слов
Разделение слов 3
Обертка слов
Добавить номера строк
Добавить префиксы строк
Добавить Суффиксы строк
Добавление префикса и суффикса
Поиск самой длинной текстовой строки
Поиск самой короткой текстовой строки
Удаление повторяющихся строк
Удаление пустых строк
Рандомизатор текстовых строк
Рандомизатор букв
Соединение текстовых строк
Разделитель строк
Реверсивное преобразование текстовых строк
Фильтр текстовых строк
Количество букв в счетчике текста
Количество слов в 900 0002 Количество строк в Счетчик текста
Счетчик количества абзацев в тексте
Калькулятор частоты букв
Калькулятор частоты слов
Калькулятор частоты фраз
Статистика текста
Средство выбора случайных элементов
Генератор случайных JSON
Генератор случайных XML
Генератор случайных YAML
Генератор случайных CSV
Генератор случайных паролей
2 Генератор случайных паролей
03
Генератор случайных строк
Генератор случайных чисел
Генератор случайных дробей
Генератор случайных бинов
Генератор случайных чисел
Генератор случайных чисел
Генератор случайных шестнадцатеричных чисел
Генератор случайных байтов
Генератор случайных IP-адресов
Генератор случайных MAC-адресов
Генератор случайных UUID
Генератор случайных GUID
Генератор случайных дат
3
2 Генератор случайных чисел Генератор
Генератор чисел Фибоначчи
Генератор числа Пи
E Генератор цифр
Преобразователь десятичных чисел в научные
Преобразователь научных чисел в десятичные
Преобразователь JPG в PNG
Конвертер PNG в JPG
Конвертер GIF в PNG
Конвертер GIF в JPG
Конвертер BMP в PNG
Конвертер BMP в JPG
Конвертер изображения в Base64 4 Преобразователь
Преобразователь XML в Base64
Преобразователь Hex в RGB
Преобразователь RGB в Hex
Преобразователь CMYK в RGB
Преобразователь RGB в CMYK
Преобразователь CMYK в Hex
Конвертер Hex в CMYK
Кодировщик IDN
Декодер IDN
Конвертер миль в километры
Конвертер километров в мили
Конвертер градусов Цельсия в градусы Цельсия 002 Конвертер градусов в радианы
Конвертер градусов в радианы
Конвертер фунтов в килограммы
Конвертер килограммов в фунты
Мой IP-адрес
Все инструменты
Совет: вы можете использовать аргумент запроса ?input=text для передачи текста в инструменты.
Обратная польская запись — RPN
Поиск инструмента
Найдите инструмент в dCode по ключевым словам:
Просмотрите полный список инструментов dCode
Обратная польская нотация
Инструмент для записи в обратной польской нотации RPN: нотация post-pifex, позволяющая записывать математические выражения без скобок.
Результаты
Обратная польская запись — dCode
Теги: Арифметика, Система записи
Поделиться
dCode и другие
dCode бесплатен, а его инструменты являются ценным подспорьем в играх, математике, геокэшинге, головоломках и задачах, которые нужно решать каждый день! Предложение ? обратная связь? Жук ? идея ? Запись в dCode !
Написание обратной польской записи
Математическое выражение для записи в РПН
См. также: Калькулятор
Калькулятор обратной польской записи
Выражение RPN для вычисления
Ответы на вопросы (FAQ)
Что такое обратная польская запись? (Определение)
Обратная польская нотация (RPN), также называемая постфиксной нотацией, представляет собой математическую нотацию арифметических выражений, в которой операнды (числа) записываются перед операторами (+, -, *, /), избегая использования круглых скобок. RPN в первую очередь адаптирован для технического компьютера / электронного использования и имеет особенность избегать использования круглых скобок.
Как писать в обратной польской записи?
В нотации NPI операнды отображаются перед операторами. Эта нотация подразумевает сначала запись операндов, а затем порядок операций.
Пример: a × (b + c) записывается a b c + ×
Алгоритм преобразования в RPN называется методом маневровой станции.
Кто использует обратную польскую запись?
Нотация имеет то преимущество, что в ней больше не используются круглые скобки, что снижает количество ошибок, а иногда она выполняется быстрее, чем обычное вычисление. RPN также используется в некоторых языках программирования, таких как Forth, а также в калькуляторах HP и в некоторых научных вычислениях, где выигрыш и время вычислений могут быть интересны.
Исходный код
dCode сохраняет за собой право собственности на исходный код «Reverse Polish Notation». За исключением явной лицензии с открытым исходным кодом (указано Creative Commons / бесплатно), алгоритма «Обратной польской нотации», апплета или фрагмента (конвертер, решатель, шифрование/дешифрование, кодирование/декодирование, шифрование/дешифрование, транслятор) или «Обратной польской нотации». Notation» (вычисление, преобразование, решение, расшифровка/шифрование, расшифровка/шифрование, декодирование/кодирование, перевод), написанные на любом информационном языке (Python, Java, PHP, C#, Javascript, Matlab и т. д.) и загрузка всех данных, script или доступ к API для «обратной польской нотации» не являются общедоступными, то же самое для автономного использования на ПК, мобильных устройствах, планшетах, iPhone или в приложениях для Android! Напоминание: dCode можно использовать бесплатно.
Цитировать dCode
Копирование и вставка страницы «Обратная польская нотация» или любых ее результатов разрешена, если вы цитируете dCode! Экспорт результатов в виде файла .
AL-AL2O3-AL2(SO4)2-AL(OH)3-AL2O3-AL(NO3)3 — вопрос №2583026 — Учеба и наука
Ответы
11. 09.17
Елена Васильевна
Читать ответы
✔Олеся / Математика
Читать ответы
Михаил Александров
Читать ответы
Посмотреть всех экспертов из раздела Учеба и наука
Похожие вопросы
Решено
Точка движется в плоскости XOY. Вектор ŕ, модуль которого равен 1м, направлен под углом 30° к оси X. Чему равны проекции вектора ŕ на оси X и Y?
Помогите пожалуйста! Важно само решение, а не ответ.
С помощью рисунков или фотографий с подписями вставь рассказ на тему «Мы – разные, мы – вместе!». В подписях к иллюстрациям отрази, какие события
Построить график функции y=2x-2 и определить проходит ли график через точку:A(10;-20)
Запишите в виде суммы разрядных слагаемых числа
КОТ МАТРОСКИН ВЫРАСТИЛ В САДУ 246 КГ ЯБЛОК А И 354 КГ ГРУШ 6 ЧАСТЬ ВСЕХ ФРУКТОВ ОН ОТДАЛ СВОИМ ДРУЗЬЯМ А5 ЧАСТЬРОДНЫМ СКОЛЬКО КГ ОТДАЛ
Пользуйтесь нашим приложением
Сколько граммов сульфата алюминия Al2(SO4)3 будет получено при полной реакции 270 г Al
Выберите область веб-сайта для поиска
ХимияВсеУчебные пособияПомощь в выполнении домашних заданийПланы уроков
Искать на этом сайте
Цитата страницы
Начать эссе
значок-вопрос
Задайте вопрос
Начать бесплатную пробную версию Скачать PDF
PDF
Цитата страницы
Цитировать
Поделиться ссылкой
Делиться
Укажите эту страницу следующим образом:
«Сколько граммов сульфата алюминия Al2(SO4)3 будет получено в результате полной реакции 270 г Al» eNotes Editorial , 7 октября 2013 г. , https://www.enotes.com/homework-help/how -много-граммов-сульфата-алюминия-al2-so4-3-will-458451.
По состоянию на 31 мая 2023 г.
`Al_2(SO_4)_3` образуется, когда мы добавляем кислоту `H_2SO_4` к твердому Al.
Мольное соотношение
Al:Al_2(SO_4)_3 = 2:1
Молярная масса Al = 27 г/моль
Количество прореагировавшего Al = 270/27 = 10 моль
Количество Ал_2( SO_4)_3` образовалось `= 1/2xx10 = 5 моль`
Молярная масса `Al_2(SO_4)_3` = 342 г/моль
Масса полученного `Al_2(SO_4)_3` `= 342xx5 = 1710 г`
Таким образом, мы можем получить максимум 1710 г `Al_2(SO_4)_3` .
Предположение
Серной кислоты достаточно для завершения реакции
См. eNotes без рекламы
Начните с 48-часовой бесплатной пробной версией , чтобы получить доступ к более чем 30 000 дополнительных руководств и более чем 350 000 вопросов помощи при выполнении домашних заданий, на которые наши эксперты ответили.
Получите 48 часов бесплатного доступа
Уже зарегистрированы? Войдите здесь.
Дополнительное чтение
https://en.wikipedia.org/wiki/Алюминий
https://en.wikipedia.org/wiki/Алюминий_сульфат
Утверждено редакцией eNotes Задайте вопрос
Похожие вопросы
Просмотреть все
Химия
Последний ответ опубликован 01 апреля 2011 г. в 12:27:06.
Последний ответ опубликован 12 июля 2012 г. в 7:49.:28 вечера
Рассчитайте процентный состав C14H9Cl5 (ДДТ).
2 Ответы воспитателя
Химия
Последний ответ опубликован 05 февраля 2012 г. в 11:41:27.
. Откуда известна полнота отделения бензойной кислоты от хлористого натрия при добавлении раствора AgNO3? Опыт был по кристаллизации. этот вопрос основан на…
1 Ответ воспитателя
Чо как чот Аль; Al2O3; Al2(SO4)3; Zn(ОН)2; ZnO; Nh5HCO3; Nh5h3PO4
Кау Хой:
08. 03.2022
6 736
Cho các chất: Al; Ал 2 О 3 ; Ал 2 (СО 4 ) 3 ; Zn(ОН) 2 ; ZnO; НХ 4 ОХС 3 ; NH 4 H 2 PO 4 ; НаХС; KHCO 3 ва (NH 4 ) 2 CO 3 . Số chất vừa phản ứng với dung dịch NaOH, vừa phản ứng với dung dịch HCl là:
А. 6
Đáp án chính xác
Xem lời giải
Câu hỏi trong đề: Cac dạng bài tập Sự điện li cơ bản, nâng cao có lời giải !!
Куанг Као
Đáp án B
Các chất đó là: Al; Ал 2 О 3 ; Zn(ОН) 2 ; ZnO; NH 4 HCO 3 ; NH 4 H 2 ПО 4 ; НаХС; KHCO 3 ва (NH 4 ) 2 CO 3 .
CÂU HỎI HOT CÙNG CHỦ ĐỀ
Câu 1:
Dung dịch A có pH < 7, tác dụng được với dung dịch Ba (NO 3 ) 2 tạo kết tủa màu trắng. Тим Дунг Дох A:
А. HCl
Б. Нет данных 2 ТЗ 4
C. H 2 SO 4
D. Na 2 CO 3
Ксем Джап »
08. 03.2022
16 552
Кау 2:
Cho các chất: Al(OH) 3 ; NaAlO 2 ; Ал 2 О 3 ; Zn(ОН) 2 ; Мг(ОН) 2 ; ZnO; СО 2 ; NaHCO 3 ; K 2 HPO 4 ; КХС; ХСО 3 ; Na 2 ZnO 2 ; AgNO 3 và Fe(OH) 3 . Có mấy chất trong dãy có tính chất lưỡng tính:
А. 7
Б. 8
С. 9
Д. 10
Ксем Джап »
08.03.2022
13 895
Кау 3:
Dãy nào vừa phản ứng với dung dịch HCl, vừa phản ứng với dung dịch NaOH:
А. Al(OH) 3 ; (NH 2 ) 2 CO; NH 4 Cl; ZnO
Б. NaHCO 3 ; Zn(OH) 2 ; Al(OH) 3 ; CH 3 COONH 4
С. Ba(OH) 2 ; AlCl 3 ; ZnO; NaHCO 3
D. Mg(HCO 3 ) 2 ; FeO; КОН ; Кр 2 О 3
Ксем Джап »
08.03.2022
8 569
Кау 4:
Cho các chất: NaCl; AlCl 3 ; CuSO 4 ; HCl; AgNO 3 ; Ва(ОН) 2 . Có mấy chất có môi trường axit:
А. 4
Б. 3
С. 2
Д. 1
Ксем Джап »
08.03.2022
7 766
Кау 5:
Cho các chất: phèn K – Al; С 2 Н 5 ОН ; глюкоза; сахароз; тин бут; Ду Ан; CH 3 COOH; HCOOCH 3 ; CH 3 CHO; С 3 Н 6 ; Ca(OH) 2 và CH 3 COONH 4 ; NaHCO 3 ; KAlO 2 ; С 2 Н 4 (ОН) 2 ; Phèn amoni – sắt . Số chất điện li la:
А. 6
Б. 7
К. 9
Д. 8
Ксем Джап »
08.03.2022
6 536
Кау 6:
Tìm dung dịch B có pH > 7, tác dụng được với dung dịch K 2 SO 4 tạo kết tủa
Перетащите сюда файлы Максимальный размер файла 50МБ (хотите больше?)
Как мои файлы защищены?
Шаг 2. Преобразуйте файлы в
Convert To
Или выберите новый формат
Шаг 3 — Начать преобразование
И согласиться с нашими Условиями
Эл. адрес?
You are attempting to upload a file that exceeds our 50MB free limit.
You will need to create a paid Zamzar account to be able to download your converted file. Would you like to continue to upload your file for conversion?
* Links must be prefixed with http or https, e. g. http://48ers.com/magnacarta.pdf
Ваши файлы. Ваши данные. Вы в контроле.
Бесплатные преобразованные файлы надежно хранятся не более 24 часов.
Файлы платных пользователей хранятся до тех пор, пока они не решат их удалить.
Все пользователи могут удалять файлы раньше, чем истечет срок их действия.
Вы в хорошей компании:
Zamzar конвертировал около 510 миллионов файлов начиная с 2006 года
NUMBERS (Document)
Расширение файла
.numbers
Категория
Document File
Описание
Файлы с расширением .numbers создаются приложением Numbers для Apple, которое является частью офисного пакета Apple iWork — программного комплекса, работающего под управлением операционных систем Mac OS X и iOS, в который также входят приложения Keynote (для презентаций) и Pages (для обработки текстовых документов).
Приложение Numbers версии 1.0 для OS X было впервые представлено 7 августа 2007 года, при этом файлы .numbers позволяли выполнять множество стандартных функций для работы с таблицами, включая поддержку строк, колонок, графиков, формул и диаграмм. Версия приложения Numbers для iPad была опубликована в январе 2010 года, а самой актуальной версией приложения Numbers для OS X, является версия 3.5, релиз которой состоялся в ноябре 2014 года в рамках обновления пакета iWork 14 от Apple.
По аналогии с другими продуктами пакета iWork, приложение Numbers включает в себя набор различных раскладок, разработанных профессиональными иллюстраторами, а также поддерживает импорт документов, созданных при помощи офисного пакета Microsoft Office (в данном случае, импорт таблиц Excel).
Действия
NUMBERS Converter
View other document file formats
Технические детали
Файл с расширением . numbers представляет собой сжатый ZIP-файл, состоящий из нескольких файлов, скомбинированных специальным образом и формирующих итоговый файл с расширением .numbers. Самая последняя версия файлов приложения Numbers сохраняется с созданием директории Index, содержащей файлы в собственном формате IWA, директорию Metadata с файлами в формате .plist и одними или несколькими JPG-изображениями, позволяющими производить быстрый просмотр. Также как и файлы, созданные в программных продуктах от Microsoft, файлы .numbers страдают от слабой совместимости с большинством других лидирующих программных решений для работы с таблицами.
Ассоциированные программы
Apple iWork
Numbers 2.1
Numbers 2.2
Numbers 2.3
Numbers 3.0
Numbers 3. 5
Разработано
Apple
Тип MIME
application/x-iwork-numbers-sffnumbers
application/vnd.apple.numbers
Полезные ссылки
Обзор приложения Numbers 3.5 от MacWorld
Обзор пакета iWork 14 от Ars Technica
Форматы файлов, поддерживаемые Numbers
Более подробная информация о Numbers
Официальная информация об Apple iWork Numbers
XLS (Document)
Расширение файла
. xls
Категория
Document File
Описание
«Microsoft Excel» является коммерческим приложением электронных таблиц, написанным и распростроняемым «Microsoft» для «Microsoft Windows» и «Mac OS X». Версии «Excel» до 2007 года для сохранения файлов используют формат XLS. В него входят вычисления, графические инструменты, сводные таблицы и макро язык программирования «Visual Basic» для приложений. Он стал широко используемой электронной таблицей на данных платмормах, особенно после 5 версии в 1993 году, а также он почти полностью заменил «Lotus 1-2-3» в качестве стадарта индустрии для электронных таблиц. «Excel» является частью «Microsoft Office». Последние версии – 2010 для «Microsoft Windows» и 2011 для «Mac OS X».
Действия
XLS Converter
View other document file formats
Технические детали
До 2007 года «Microsoft Excel» использовал собственный бинарный формат файлов, называемый «Binary Interchange File Format (BIFF)» в качестве главного формата. Используемый в качестве основы для XLS-файлов это постоянный формат, который поддерживает авторинг и манипулирование содержанием рабочих книг и шаблонов рабочих книг. Большинство версий «Microsoft Excel» могут читать форматы CSV, DBF, SYLK, DIF и др.
Ассоциированные программы
Microsoft Excel
Microsoft Excel Viewer
OpenOffice
Разработано
Microsoft
Тип MIME
application/vnd.ms-excel
Полезные ссылки
Подробнее о формате XLS
Как отркыть файл XLS без «Microsoft Excel»
Спецификация бинарного формата файлов «Microsoft Office»
Преобразование файлов NUMBERS
Используя Zamzar можно конвертировать файлы NUMBERS во множество других форматов
numbers в csv
(Comma Separated Values)
numbers в numbers09 (Apple iWork ’09 Numbers Spreadsheet)
numbers в pdf
(Portable Document Format)
numbers в xls
(Microsoft Excel Spreadsheet)
numbers в xlsx
(Microsoft Excel 2007 Spreadsheet)
NUMBERS to XLS — Convert file now
Available Translations: English
| Français
| Español
| Italiano
| Pyccĸий
| Deutsch
Преобразуйте NUMBERS в EXCEL онлайн бесплатно
редактор
Зритель
Преобразование
Слияние
Разблокировать
Защищать
Сплиттер
Сравнение
Аннотация
Парсер
Метаданные
Водяной знак
Поиск
Заменять
Повернуть
Обеспечить регресс
Диаграмма
Ипотека
Сборка
Перевод
Компресс
Прозрачный
ИМТ
ВебКонвертер
Питаться от
aspose. com
&
aspose.cloud
Перетащите или загрузите свои файлы
Введите адрес
*Загружая свои файлы или используя наш сервис, вы соглашаетесь с нашими
Условия использования
&
политика конфиденциальности
Популярные конвертеры:
NUMBERS to MD
NUMBERS to FODS
NUMBERS to XLSM
NUMBERS to SVG
NUMBERS to TABDELIMITED
Ваши файлы успешно обработаны
СКАЧАТЬ СЕЙЧАС
Сохранить в облачное хранилище:
Отправить по электронной почте
On Premise API
Нажмите Ctrl+D, чтобы сохранить его в закладках, чтобы не искать его снова
Поделиться через фейсбук
Поделиться в Твиттере
Посмотреть другие приложения
Попробуйте наш облачный API
См. исходный код
Оставить отзыв
Добавить это приложение в закладки
Нажмите Ctrl + D, чтобы добавить эту страницу в избранное, или Esc, чтобы отменить действие.
Вы хотите сообщить об этой ошибке на форум, чтобы мы могли изучить ее и решить проблему? Вы получите уведомление по электронной почте, когда ошибка будет исправлена.
Email:
Сделайте этот форум закрытым, чтобы он был доступен только вам и нашим разработчикам.
Вы успешно сообщили об ошибке. Вы получите уведомление по электронной почте, когда ошибка будет исправлена.
Нажмите эту ссылку, чтобы посетить форумы.
Вы уверены, что хотите удалить файлы?
Обработка…
Numbers to Excel — конвертируйте свои NUMBERS в XLS бесплатно онлайн
Конвертируйте NUMBERS в XLS онлайн и бесплатно
Шаг 1. Выберите файлы для конвертации
Перетаскивание файлов Макс. размер файла 50MB (хотите больше?)
Как мои файлы защищены?
Шаг 2. Конвертируйте ваши файлы в
Конвертировать в
Или выбрать другой формат
Шаг 3. Начать конвертацию
(и принять наши Условия)
Электронная почта, когда закончите?
Вы пытаетесь загрузить файл, размер которого превышает наш свободный лимит в 50 МБ.
Вам нужно будет создать платную учетную запись Zamzar, чтобы иметь возможность скачать преобразованный файл. Хотите продолжить загрузку файла для конвертации?
* Ссылки должны иметь префикс http или https , например. http://48ers.com/magnacarta.pdf
Частные лица и компании доверяют Zamzar с 2006 года. Мы обеспечиваем безопасность ваших файлов и данных и предлагаем выбор и контроль над удалением файлов.
Свободно конвертированные файлы надежно хранятся не более 24 часов
Файлы платных пользователей хранятся до тех пор, пока они не решат их удалить
Все пользователи могут удалять файлы до истечения срока их действия
Попробовала и сразу влюбилась! Это было так легко использовать! После пары преобразований я купил ребятам чашку кофе. Еще пара и решил, что это слишком хорошо, чтобы злоупотреблять! Я присоеденился! Моя жизнь намного проще!
Тилли
У меня был огромный проблемный файл для преобразования, который не мог пройти обычный процесс автоматического преобразования. Команда Zamzar быстро отреагировала на мою просьбу о помощи и предприняла дополнительные шаги, необходимые для того, чтобы сделать это вручную.
ПДинСФ
Использовал его более года для преобразования моих банковских выписок в файлы csv. Отличное быстрое приложение, значительно увеличило мою производительность. Также замечательная поддержка — всегда быстро помогали!
Агата Вежбицкая
Я использовал этот продукт в течение многих лет. И обслуживание клиентов отличное. Только что возникла проблема, когда мне предъявили обвинение, и я не согласился с обвинением, и они позаботились об этом, хотя в этом не было необходимости.
JH
Я был так благодарен Замзару за поддержку с начала пандемии до наших дней. Их обслуживание является первоклассным, и их готовность помочь всегда на высоте.
Мэри
Очень полезный и профессиональный сайт. Сервис прост в использовании, а администраторы услужливы и вежливы.
Дэвид Шелтон
Я использую Zamar всякий раз, когда мне нужно преобразовать аудио- и видеофайлы из нескольких отправителей в единый формат файла для редактирования аудио и видео. Я могу сделать несколько больших файлов за короткий промежуток времени.
Кристофер Би
Отлично подходит, когда вам нужно много конверсий за короткое время. Вы имеете прямой доступ и даже можете оформить подписку всего на месяц.
Сабина Калис
Большое спасибо всем вам за помощь в правильном преобразовании СТАРЫХ файлов. 20 лет, довольно долгий срок, просмотр файлов навевает мне много воспоминаний. Это лучший подарок, который я получил в прошлом году. Спасибо всем еще раз.
Цзюнн-Ру Лай
Фантастический сервис! Компьютер моей мамы умер, и у нее есть более 1000 файлов Word Perfect, которые она по какой-то причине хочет сохранить. Поскольку Word Perfect практически мертв, я решил конвертировать все ее файлы. Преобразователь Замзара был идеальным.
Арон Бойетт
Нам доверяют сотрудники этих брендов
Сотрудники некоторых из самых известных мировых брендов полагаются на Zamzar для безопасного и эффективного преобразования своих файлов, гарантируя, что у них есть форматы, необходимые для работы. Сотрудники этих организаций, от глобальных корпораций и медиа-компаний до уважаемых учебных заведений и газетных изданий, доверяют Zamzar предоставление точных и надежных услуг по конвертации, в которых они нуждаются.
Ваши файлы в надежных руках
От вашего личного рабочего стола до ваших бизнес-файлов, мы обеспечим вас
Мы предлагаем ряд инструментов, которые помогут вам конвертировать ваши файлы наиболее удобным для вас способом. Помимо нашей онлайн-службы преобразования файлов, мы также предлагаем настольное приложение для преобразования файлов прямо с вашего рабочего стола и API для автоматического преобразования файлов для разработчиков. Какой инструмент вы используете, зависит от вас!
Хотите конвертировать файлы прямо с рабочего стола?
Получить приложение
Полностью интегрирован в ваш рабочий стол
Преобразование более 150 различных форматов файлов
Конвертируйте документы, видео, аудио файлы в один клик
Нужна функциональность преобразования в вашем приложении?
Изучите API
Один простой API для преобразования файлов
100 форматов на ваш выбор
Документы, видео, аудио, изображения и многое другое. ..
Инструменты для преобразования ваших файлов
В Zamzar вы найдете все необходимые инструменты для преобразования и сжатия в одном месте. С поддержкой более 1100 типов преобразования файлов, независимо от того, нужно ли вам конвертировать видео, аудио, документы или изображения, вы легко найдете то, что вам нужно, и вскоре ваши файлы будут в форматах и размерах, которые вам подходят.
ЦИФРЫ Формат документа
Конвертер чисел
Numbers — это программа Apple для работы с электронными таблицами, эквивалентная таким программам, как Microsoft Excel. Он входит в стандартную комплектацию операционных систем Apple iOS, iPadOS и macOS как часть пакета Apple iWork. Это означает, что полное приложение Numbers автоматически устанавливается на iPhone, iPad и Mac. Многие люди с устройствами Apple используют Numbers, потому что они могут создавать, просматривать и редактировать электронные таблицы на своих устройствах Apple без дополнительной подписки или покупки программного обеспечения.
Numbers позволяет пользователям создавать электронные таблицы с нуля или из шаблонов, вводя данные в ячейки. Вы также можете использовать его для расчетов и сводных таблиц, а также для создания диаграмм и графиков. Поскольку это формат Apple, файлы NUMBERS не открываются напрямую в Microsoft Excel или других программах для работы с электронными таблицами. Однако вы можете использовать программу Numbers или учетную запись iCloud для сохранения электронных таблиц в формате CSV, XLS или XLSX.
Связанные инструменты
Конвертеры документов
Конвертер чисел
Формат документа XLS
XLS-конвертер
Формат файла XLS был разработан Microsoft для своей программы электронных таблиц Excel. XLS — это двоичный формат электронной таблицы, который может включать данные ячеек и формулы, а также диаграммы и макросы. Excel — одна из самых популярных программ для записи, организации, расчета и анализа данных. Он часто используется для составления бюджета и отчетности.
Microsoft представила новый формат XLSX для Excel в 2007 году, чтобы заменить XLS. Однако вы по-прежнему можете открывать файлы XLS в более новых версиях Excel, а также сохранять файлы XLSX в этом более старом формате. Excel является частью пакета Microsoft Office, который доступен как разовая покупка программного обеспечения или подписка на Office 365. Другие программы для работы с электронными таблицами, которые обычно могут открывать файлы XLS, включают Google Sheets, Apple Numbers и LibreOffice Calc.
Связанные инструменты
Конвертеры документов
XLS-конвертер
Как преобразовать NUMBERS в файл XLS?
1. Выберите файл NUMBERS, который вы хотите преобразовать.
2. Выберите XLS в качестве формата, в который вы хотите преобразовать файл NUMBERS.
3. Нажмите «Преобразовать», чтобы преобразовать файл NUMBERS.
Преобразование из ЦИФРОВ
Используя Zamzar, можно конвертировать файлы NUMBERS во множество других форматов:
NUMBERS в CSV
НОМЕРА до НОМЕРА09
НОМЕРА в PDF
НОМЕРА в XLS
НОМЕРА до XLSX
Преобразование в ЦИФРЫ
Используя Zamzar, можно конвертировать множество других форматов в файлы NUMBERS:
XLS в НОМЕРА
XLSX на НОМЕРА
Online NUMBERS to XLS Converter
Вы также можете конвертировать NUMBERS во многие другие форматы файлов.
Функция — округление x в меньшую сторону (пример floor(4.5)==4.0)
ceiling(x)
Функция — округление x в большую сторону (пример ceiling(4.5)==5.0)
sign(x)
Функция — Знак x
erf(x)
Функция ошибок (или интеграл вероятности)
laplace(x)
Функция Лапласа
asech(x)
Функция — гиперболический арксеканс от x
csch(x)
Функция — гиперболический косеканс от x
sech(x)
Функция — гиперболический секанс от x
acsch(x)
Функция — гиперболический арккосеканс от x
Постоянные:
pi
Число «Пи», которое примерно равно ~3. 14159..
e
Число e — основание натурального логарифма, примерно равно ~2,7183..
i
Комплексная единица
oo
Символ бесконечности — знак для бесконечности
Исчисление
— Показать, что $\log(x+1)-\log(x)0$
Задавать вопрос
спросил
Изменено
4 года назад
Просмотрено
813 раз
$\begingroup$
Покажите, что $\log(x+1)-\log(x)<\frac{1}{x}$ при $x >0$.
Мне сказали, что это очень просто, но я не знаю, как это решить. Любая помощь будет здорово.
исчисление
неравенство
логарифмы
$\endgroup$
$\begingroup$
Функция $\log(1+t)$ строго вогнута, поэтому ее график остается ниже касательной в точке $0$: для любых $t\not=0$ и $t>-1$
$$\log(1+t)< t. $$
Ваше неравенство эквивалентно
$$\log(x+1)-\log(x)=\log\left(1+\frac1x\right)< \frac1x.$$
$\endgroup$
2
$\begingroup$
Применить теорему о среднем значении: если $f$ дифференцируема на $]a,b[$ и непрерывна на $[a,b]$, то
$$(\exists c\in]a,b[)\,f(b)-f(a)=(b-a)f'(c)$$
Так вот :
$$\ln(x+1)-\ln(x)=(x+1-x)\ln'(c)=\frac1c< \frac1x$$
потому что $x+1>c>x$.
Rmk : вы также можете изучить функцию $f:x\mapsto \ln(x+1)-\ln(x)-\frac1x$…
$\endgroup$
$\begingroup$
Рассмотрим $f(x)=\log\left(\frac{x+1}{x}\right)=\log\left(1+\frac{1}{x}\right)$ . Установите $u=1/x$, и тогда, сделав хорошую цифру, вы увидите, что $\log(1+u)\le u$ и все готово.
$\endgroup$
$\begingroup$
Пусть $x>0$.
$f: t\mapsto \ln(t)$ непрерывен в $[x,x+1]$ и дифференцируем в $(x,x+1)$, таким образом, согласно MVT,
Изменено
1 год, 6 месяцев назад
9x$ для $x\gt 0$ тогда я могу взять $\log$..
реальный анализ
$\endgroup$
9
$\begingroup$
Пусть $$f(x) = \ln(1+x)-x\;,$$ Где $x>0$
Теперь $$\displaystyle f'(x) = \frac{1}{1 +x}-1 = -\frac{x}{1+x}<0$$ для всех $x>0$
Итак, функция $f(x)$ является строго убывающей функцией
Итак, здесь $$x> 0\Rightarrow f(x) 9х > 1, \ 1 + х > 1$$
Теперь, когда мы логарифмируем обе части, знак неравенства не меняется.
$\endgroup$
$\begingroup$
Пусть $f: x \mapsto x — \log (1+x)$ на $]0, +\infty[$. Так как $f'(x) = 1 — \frac{1}{1+x} > 0$ для всех $x > 0$, то $f$ строго возрастает на $]0, +\infty[$. Сможешь закончить?
$\endgroup$
$\begingroup$
Функция $\ln (1+x)$ строго вогнута на $(-1,\infty).$ Таким образом, эта функция лежит ниже любой из своих касательных в этой области (кроме точки касания). Поскольку $y=x$ — это прямая, касающаяся $y=\ln (1+x)$ в точке $(0,0),$, мы получили желаемый результат (и даже больше).
$\endgroup$
$\begingroup$
Ваше неравенство можно улучшить. $\exp(x)$ строго выпукла.
Функция — округление x в меньшую сторону (пример floor(4.5)==4.0)
ceiling(x)
Функция — округление x в большую сторону (пример ceiling(4.5)==5.0)
sign(x)
Функция — Знак x
erf(x)
Функция ошибок (или интеграл вероятности)
laplace(x)
Функция Лапласа
asech(x)
Функция — гиперболический арксеканс от x
csch(x)
Функция — гиперболический косеканс от x
sech(x)
Функция — гиперболический секанс от x
acsch(x)
Функция — гиперболический арккосеканс от x
Постоянные:
pi
Число «Пи», которое примерно равно ~3. 14159..
e
Число e — основание натурального логарифма, примерно равно ~2,7183..
i
Комплексная единица
oo
Символ бесконечности — знак для бесконечности
Чем подкормить рассаду, чтобы она окрепла
15 апреля 2022
Ликбез
Советы
Присмотритесь к побегам. Возможно, пришло время подкормить их.
Как понять, что рассаде нужны удобрения
Если вы позаботились о качестве почвы при пикировке своих растений, то о дополнительной подкормке можно не беспокоиться. Но, к сожалению, садоводы не всегда выбирают хороший грунт, так что дефицит полезных веществ может отразиться на состоянии рассады. Если рост побегов остановился или замедлился, листья изменили цвет, а стебли вытянулись и стали тонкими, самое время подкормить её.
Чем подкормить рассаду
Подкормка бывает корневой и внекорневой. В первом случае речь идёт про полив растения под корень, во втором — про опрыскивание.
Подкармливать рассаду можно минеральными и органическими удобрениями. Или выбрать комплексные средства — они бывают универсальными и специальными для разных культур. Этот вариант подойдёт, если нет времени или желания разбираться во всех тонкостях подкормок.
Также на пользу пойдут различные стимуляторы роста и укоренители.
Минеральные удобрения
Основные вещества, в которых нуждаются растения в течение жизни, — это азот, фосфор и калий. Недостаток любого из них сказывается на качестве и внешнем виде рассады. В зависимости от проблемы саженцы можно подкармливать этими элементами по отдельности. Главное — понять, за что они отвечают.
1. Азотные удобрения
Азот отвечает за обмен веществ в клетках растений. Он помогает выработке хлорофилла и белков, обеспечивает гармоничный рост и наращивание зелёной массы.
Признаки нехватки азота:
медленный рост или его остановка;
жёлтые или бледные листья;
тонкие и слабые стебли.
Есть множество удобрений с азотом: аммиачная вода, аммиачная селитра, мочевина, сернокислый аммоний, сульфат аммония. У каждого свои особенности применения. Поэтому перед подкормками внимательно прочитайте инструкцию на упаковке.
Например, аммиачная селитра, сульфат аммония и мочевина обычно продаются в виде гранул и порошка, но лучше усваиваются растением в жидкой форме. Для приготовления раствора возьмите 3–4 г удобрения на 1 л воды. Полученной смесью полейте растения, а через 1–2 часа аккуратно разрыхлите верхний слой почвы.
2. Фосфорные удобрения
Фосфор помогает развитию корневой системы растений, а также повышает их устойчивость к низким температурам и засухам.
Признаки нехватки фосфора:
потемнение и фиолетово-багровая окраска листьев и стеблей;
тёмный, а иногда почти чёрный цвет засохшей листвы.
Вариантов удобрений тут тоже много: аммофос, двойной суперфосфат, диаммофос, костная мука, метафосфат калия, простой суперфосфат, фосфоритная мука. Выбрать можно что угодно. Только надо помнить, что костная и фосфоритная мука практически не растворяется в воде, поэтому ей лучше подкармливать непосредственно перед посадкой растений в открытый грунт или теплицу.
Дома же целесообразнее использовать водорастворимые удобрения. Например, 1 г двойного суперфосфата залить 1 л горячей воды, дать остыть и подкормить рассаду под корень. Можно повторить процедуру через 7–10 дней.
3. Калийные удобрения
Калий — важный участник процесса фотосинтеза, поэтому его дефицит в первую очередь отражается на листьях. Он также отвечает за иммунитет растения, обильное цветение и вкус плодов.
Признаки нехватки калия:
краевой ожог листьев;
усыхание и опадание листьев.
Подкормить рассаду можно калийной селитрой, калимагнезией, монофосфатом калия, сульфатом калия, сернокислым калием. Например, растворите 0,7–1 г монофосфата калия в 1 л воды и полейте рассаду под корень. Спустя 10–15 дней подкормку можно повторить.
Органические удобрения
Коровяк и птичий помёт — натуральные подкормки. Они обеспечат рассаду азотом, калием, магнием, фосфором, кальцием, марганцем и другими полезными веществами. К тому же их можно комбинировать с минеральными удобрениями.
Хотя есть и явный недостаток — стойкий запах. Перед тем как поливать растения, и коровяк, и помёт надо залить водой и настаивать несколько дней в тепле. В условиях квартиры это может стать настоящей проблемой, поэтому можно купить гранулированный навоз. Он не имеет резкого запаха и просто вносится в почву в сухом виде.
Ещё один вариант — концентрат биогумуса, который содержит комплекс питательных веществ, микро- и макроэлементов. Это средство продаётся в полуготовом виде, так что его можно просто разбавить водой и настаивать всего несколько часов.
Комплексные удобрения
Эти средства — источник полезных веществ для растений. При выборе стоит обратить особое внимание на состав и пропорции компонентов.
Трёхкомпонентные удобрения. Содержат сразу азот, фосфор и калий. Это аммофоска, диаммофоска, нитроаммофоска, нитрофоска.
Все они относятся к сложным, так как при производстве вещества не просто механически миксуются, а вступают в химические реакции. В состав могут также входить сера, магний и другие элементы.
Биостимуляторы (стимуляторы роста и укоренители)
Эти средства тоже содержат целый комплекс веществ, которые нужны растениям. Плюс тут есть фунгициды, которые защищают от грибковых болезней.
К биостимуляторам относится, например, янтарная кислота. С ней растения становятся выносливее, лучше развиваются и усваивают питательные вещества из грунта. Однако заменить удобрения она не может, а лишь дополняет их.
Продаётся янтарная кислота в виде таблеток и порошка, которые нужно разводить в воде. Для рассады достаточно двух-трёх поливов, последний из которых лучше провести за несколько дней до пересадки. Полезна она и через 2–3 дня после пикировки. В обоих случаях средство поможет рассаде укорениться.
Опрыскивание — ещё один вариант использования янтарной кислоты. Оно особенно эффективно перед пересадкой и после неё, а также для помощи повреждённым и ослабленным растениям.
Купить можно и другие специальные стимуляторы и укоренители разных брендов. Например, «Эпин» и «Циркон» помогут растениям пережить стресс после пикировки, пересадки и прищипывания, а «Корневин» простимулирует рост корневой системы.
Когда подкармливать рассаду
Не спешите подкармливать растения, которые только взошли. Это скорее навредит, чем пойдёт на пользу.
Когда подкармливать баклажаны
Первый раз внести удобрение можно, когда у растений уже есть по 2–3 настоящих листа, а второй — за неделю-полторы до пересадки на постоянное место.
Когда подкармливать капусту
Первый раз внести удобрения можно через неделю после пикировки, а второй — за 7 дней до высадки в грунт.
Когда подкармливать огурцы
Рассаду огурцов можно высаживать уже через 25–30 дней, так что при правильном грунте можно обойтись и без удобрений. А дополнительные вещества нужны только при появлении проблем.
Если без подкормки не обойтись, дождитесь, пока появятся хотя бы один-два настоящих листа, и тогда вносите удобрение в первый раз. Во второй раз огурцы можно подкормить примерно через две недели.
Когда подкармливать перец
Когда у растения появится настоящий лист, можно внести первую порцию удобрения. Время для второй подкормки наступит через две-три недели, а для третьей — за 7 дней до пересадки.
Когда подкармливать помидоры
Первую подкормку можно внести, когда у всходов появятся первые настоящие листья. Подходящее время для второй — через 12–14 дней после пикировки. Третий раз подкормить помидоры можно ещё спустя две недели, а четвёртый — за 7–10 дней до высадки в открытый грунт, теплицу или балконный ящик.
Как правильно подкормить рассаду
Есть несколько важных правил.
Соблюдайте пропорции
Перед использованием удобрения внимательно прочитайте инструкцию на упаковке. В зависимости от производителя дозировка может различаться. Следуйте рекомендациям при приготовлении раствора — избыток питательных веществ может погубить растения так же, как и их нехватка.
Предварительно полейте растения
Вносить удобрения нужно во влажный грунт, чтобы не обжечь корни растения. Полейте рассаду вечером обычной водой, а утром — питательным раствором. Такая методика убережёт корневую систему от ожога, а почву — от переувлажнения.
Кроме того, позаботьтесь о правильном поливе в интервалах между подкормками. Не пересушивайте грунт и не давайте ему превратиться в болото. Это одинаково опасно для рассады.
Будьте аккуратны
Лейте раствор с удобрением строго под корень, иначе рискуете обжечь листья и стебли растения. Перед началом работы с химикатами не забудьте надеть перчатки.
Выбирайте подходящее время
Поливать желательно утром, чтобы корни не пострадали во влажном грунте при более низкой ночной температуре.
А внекорневые подкормки лучше делать вечером. Во-первых, под воздействием света полезные вещества испарятся быстрее, чем успеют подействовать, а во-вторых, листья могут обгореть.
Не смешивайте разные виды удобрений
Лучше использовать средства по отдельности, потому что их сочетание может привести к непредсказуемым результатам — нужные элементы попросту не усвоятся растениями. Если не хочется разбираться в теории, проведите подкормку комплексным минеральным удобрением с равномерным распределением азота, фосфора и калия — чаще всего это 18 : 18 : 18 или 20 : 20 : 20. Дозировку и способ применения можно найти в инструкции на упаковке.
Не подкармливайте слишком часто
Как мы уже говорили, избыток в грунте даже полезных веществ не лучший вариант для растений. Так что не стоит раз в несколько дней пытаться поить рассаду всеми подкормками, которые есть под рукой. Соблюдайте интервалы между внесениями удобрений. Кроме того, подкормки лучше начинать уже после пикировки, если вы используете качественный и богатый полезными веществами грунт.
Читайте также 🌱🌱🌱
Огород на подоконнике: как вырастить овощи, зелень и даже клубнику у себя дома
Как вырастить лаванду на подоконнике: простая инструкция
10 красивых цветов для клумбы, за которыми легко ухаживать
10 полезных товаров для создания мини-огорода на подоконнике
15 товаров для нового дачного сезона
элементарная теория чисел — Докажите, что $\sqrt 2 + \sqrt 3$ иррационально
спросил
Изменено
3 года, 4 месяца назад
Просмотрено
112 тысяч раз
$\begingroup$
Здесь уже есть ответы на этот вопрос :
Может ли $\sqrt{n} + \sqrt{m}$ быть рациональным, если ни $n,m$ не являются полными квадратами? [дубликат]
(5 ответов)
Закрыта 4 года назад.
В предыдущих упражнениях этой книги я доказал, что $\sqrt 2$ и $\sqrt 3$ иррациональны. Тогда сумма двух иррациональных чисел есть иррациональное число. Таким образом, $\sqrt 2 + \sqrt 3$ иррационально. Мой первый вопрос: правильно ли это рассуждение?
Во-вторых, книга хочет, чтобы я использовал тот факт, что если $n$ — целое число, не являющееся полным квадратом, то $\sqrt n$ иррационально. Это означает, что $\sqrt 6$ иррационально. Как нам использовать этот факт? Можем ли мы рассуждать следующим образом:
$\sqrt 6$ иррационально
$\Rightarrow \sqrt{2 \cdot 3}$ иррационально.
$\Rightarrow \sqrt 2 \cdot \sqrt 3$ иррационально
$\Rightarrow \sqrt 2$ или $\sqrt 3$ или оба иррациональны.
$\Rightarrow \sqrt 2 + \sqrt 3$ иррационально. 92 = 5 + 2 \sqrt{6}$. Но это абсурд, поскольку $\sqrt{6}$ иррационально.
$\endgroup$
2
$\begingroup$
Если $\sqrt 3 +\sqrt 2$ рационально/иррационально, то $\sqrt 3 -\sqrt 2$ также рационально, потому что $\sqrt 3 +\sqrt 2=\large \frac {1}{\sqrt 3 — \sqrt 2}$ . Теперь предположим, что $\sqrt 3 +\sqrt 2$ рационально. Если мы добавим $(\sqrt 3 +\sqrt 2)+(\sqrt 3 -\sqrt 2)$, мы получим $2\sqrt 3$, что иррационально. Но сумма двух рациональных чисел никогда не может быть иррациональной, потому что для целых чисел $a, b, c, d$ $\large \frac ab+\frac cd=\frac {ad+bc}{bd}$, что является рациональным. Следовательно, наше предположение, что $\sqrt 3 +\sqrt 2$ рационально, неверно, поэтому $\sqrt 3 +\sqrt 2$ иррационально. 92=2+2\sqrt 6 +3$ иррационально, потому что сумма иррационального и рационального ($5$) иррациональна, поэтому мы имеем противоречие.
$\endgroup$
$\begingroup$
Если вы что-нибудь знаете о теории Галуа, вот очень окольный способ доказать это (другими словами, другие ответы лучше подходят для решения этой проблемы):
$\alpha=\sqrt 2+\sqrt 3 $ — примитивный элемент расширения Галуа
$[\Bbb Q(\alpha):\Bbb Q]$ с минимальным многочленом
$(x^2-2)(x^2-3)$, который имеет степень $4$ над $\mathbb{Q}$. Отсюда следует, что $\sqrt 2+\sqrt 3\notin\mathbb Q$.
$\endgroup$
5
$\begingroup$
Сначала нужно доказать, что сумма двух иррациональных чисел дает иррациональное число. Заметьте, что это неправда. Итак, на ваш первый вопрос ваши рассуждения неверны.
$\endgroup$
$\begingroup$
Как уже отмечалось, сумма
два иррациональных числа могут быть рациональными,
так что ваше доказательство недействительно.
Это верно, даже если оба числа
положительны, как показано ниже:
Пусть $a = 0,12112111211112…$
и сформируйте $b$, заменяя каждый $1$ в $a$
до $2$ и каждые $2$ до $1$.
Итак, $b = 0,21221222122221…$
Ясно, что $a$ и $b$ иррациональны,
но $a+b = 0,33333.